Arm Muscles and Clinical Correlations, PrepU Upper Extremity questions for test 3, Anatomy Exam 2

Lakukan tugas rumah & ujian kamu dengan baik sekarang menggunakan Quizwiz!

part of flexor digitorum profundus.

The ulnar nerve innervates _______________. a. part of flexor digitorum profundus. b. part of flexor digitorum superficialis. c. extensor digitorum. d. flexor carpi radialis. e. abductor pollicis brevis.

Triquetrum

The victim of an automobile accident has a destructive injury of the proximal row of carpal bones. Which of the following bones is most likely damaged? A. Triquetrum B. Capitate C. Hamate D. Trapezium E. Trapezoid

winged scapula

Lesion of the long thoracic nerve causes _____________. A. winged scapula. B. inability to initiate abduction of the arm. C. inability to abduct the arm over 90°. D. loss of sensation over the posterior arm. E. loss of sensation over the scapula.

Supinator

Lateral epicondyle of humerus; radial collateral and anular ligaments; supinator fossa; crest of ulna, Lateral, posterior, and anterior surfaces of proximal third of radius, Supinates forearm, rotates radius to turn palm anteriorly or superiorly (if elbow is flexed), Deep branch of radial nerve (C7, C8), Radial recurrent artery

Extensor Carpi Radialis Longus

Lateral supra-epicondylar ridge of humerus, Dorsal aspect of base of 2nd metacarpal, Extend and abduct hand at the wrist joint, Radial Nerve (C6, C7), Radial artery

retraction of the scapula.

Lesion of the dorsal scapular nerve will have the greatest affect on the ____________. a. retraction of the scapula. b. abduction of the humerus. c. sensation over the shoulder. d. elevation of the clavicle. e. sensation over the anterior arm.

Flexor Digitorum Profundus

Proximal three quarters of medial and anterior surfaces of ulna and interosseous membrane, Bases of distal phalanges of 2nd, 3rd, 4th, and 5th digits, Flexes distal interphalangeal joints of digits 2, 3, 4, and 5; assists with wrist flexion, Lateral part: (to digits 2 and 3)- Median Nerve (C8, T1) (anterior interosseous branch) Medial Part (digits 4 and 4)- Ulnar nerve (C8, T1), Radial and Unlar Artery

You are treating a patient in the ED who sustained a forearm injury. You ask the patient to make the "okay" sign and instead a "pinch" posture is exhibited between the thumb and index finger. There is no flexion of the distal interphalangeal joint of the index finger and interphalangeal joint of the thumb. What nerve was affected by the accident?

anterior interosseous. The anterior interosseous nerve (branch of median) supplies the flexor pollicis longus and the lateral two tendons of the flexor digitorum profundus. Loss of these muscles results in the inability to make an "okay" sign (anterior interosseous syndrome).

Axillary nerve

A young man is brought to the emergency department after being mugged. He was stabbed in the shoulder after refusing to give his wallet to his assailant. If the stab wound lacerated the posterior humeral circumflex artery passing through the quadrangular space on the shoulder region, which of the following nerves might be injured? A. Axillary nerve B. Radial nerve C. Thoracodorsal nerve D. Suprascapular nerve E. Accessory nerve

Flexor digitorum profundus (FDP)

As part of a physical examination to evaluate muscle function in the hand, a physician holds the proximal interphalangeal joint of his patient's index finger in the extended position and instructs him to try to flex the distal interphalangeal joint, as shown below. Which of the following muscles is the doctor testing? a. Flexor digitorum profundus (FDP) b. Extensor indicis c. First lumbrical d. First dorsal interosseous e. Flexor digitorum superficialis (FDS)

25. A retired teacher suffers from a massive heart attack while playing golf and dies in the intensive care unit. Autopsy reveals the cause of death as severely diminished blood flow in the coronary arteries. This most likely resulted from embolization of an atherosclerotic plaque at the origin of which of the following vascular structures?

Ascending aorta

Anconeus

Assists triceps in extending forearm; stabilizes elbow joint; abducts ulna during pronation, radial nerve C7, C8, T1, Middle collateral branch of deep brachial artery; Recurrent interosseous artery

C

Choose the appropriate lettered structure in the radiograph of the bones of the hand (see figure). Fracture of which bone may cause paralysis of the flexor digiti minimi and opponens digiti minimi muscles?

Pronator Quadratus

Distal quarter of anterior surface of ulna, Distal quarter of anterior surface of radius, pronates forearm; deep fibers bind radius and ulna together, Anterior interosseous nerve, fro median nerve (C8, T1), Radial artery and Anterior interosseous artery

You see a 30-year-old hockey player who 'caught' a high velocity puck in his mid-sternum. Radiography clearly shows a sternal fracture. What next study might you order?

Echocardiography and ECG to see if there is pericardial effusion or other sign of cardiac injury

8. A ballet dancer falls to the floor and hurts herself during a practice session before opening night. She sustains an injury to the thoracodorsal nerve that would probably affect the strength of which of the following movements?

Extension of the arm

Opponens Digiti Minimi

Hook of Hamate and flexor retinaculum, Medial border of 5th metacarpal, Draws 5th metacarpal anterior and rotates it, bringing 5th finger into opposition with thumb, Deep branch of Ulnar Nerve (C8, T1), ulnar artery

4. Young Johnny was playing on the playground at school when he fell and struck his arm against the swing set. He ran to the school nurse, complaining of which of the following conditions as a result of injuring the radial nerve in the spiral groove of the humerus?

Inability to extend the hand

5. An indoor soccer player runs into another player while running after the ball. She falls to the ground and fractures the medial epicondyle of the humerus. Which of the following symptoms might she present with when seeing a physician in the emergency department?

Inability to sweat on the medial part of the hand

12. A 21-year-old man celebrating his birthday gets a little carried away with his friends and starts a bar fight. He is stabbed with a knife that severs the roots of C5 and C6 of the brachial plexus. Which of the following muscles is likely to be paralyzed?

Infraspinatus

21. A 78-year-old woman receives knee surgery because her lateral meniscus is torn. Before injury, the normal lateral meniscus of the knee joint:

Lies outside the synovial cavity

Abductor Digiti Minimi

Pisiform bone, Medial side of base of proximal phalanx of 5th finger, Abducts 5th finger; assists in flexion of proximal phalanx, Deep branch of Ulnar Nerve (C8, T1), ulnar artery

17. During recruitment by the local representative for the Marines, a young college student presents with the condition known as flat foot. His foot is displaced laterally and everted, and the head of the talus is no longer supported. Which of the following ligaments probably is stretched?

Plantar calcaneonavicular (spring)

Annulus fibrosis and nucleus pulposus

The insertion of artificial intervertebral disks has been attempted for patients with chronic back pain. These artificial disks are intended to replace both parts of a natural disk. What are these two parts? A. Annulus fibrosis and ligamentum flavum B. Annulus fibrosis and alar ligaments C. Annulus fibrosis and nucleus pulposus D. Centrosis and ligamentum flavum E. Centrosis and nucleus pulposus

Axillary artery

The lateral cord of the brachial plexus is named because it lies immediately lateral to which of the following structures? A. Long head of the biceps brachii muscle B. Axillary artery C. Subclavian vein D. Surgical neck of the humerus E. Pectoralis minor muscle

Biceps Tendinitis

The tendon of the long head of the biceps, enclosed by a synovial sheath, moves back and forth in the intertubercular sulcus (groove) of the humerus. Wear and tear of this mechanism can cause shoulder pain. Inflammation of the tendon (biceps tendinitis) usually is the result of repetitive microtrauma in sports involving throwing (e.g., baseball).

Which of the labelled structures in the given CT scan of the thorax indicates the left main bronchus?

The trachea, bronchi, and lungs appear as radiolucent (black) structures because of their air content. Left and right are differentiated by remembering that the conventional view of CT scans is from below, as if standing at the foot of the patient's bed and looking to the supine person's head. Also, the left main (primary) bronchus is more horizontally aligned, whereas the right main bronchus is more vertically aligned. Thus, the left main bronchus offers a more oblong profile in cross-sectional (axial) scans, while the right main bronchus is more circular

Adduction of the arm

The traditional radical mastectomy includes removal of the pectoralis major muscle. Which of the following movements is most affected postoperatively by this surgical procedure? a. Adduction of the arm b. Abduction of the arm c. Extension of the arm d. Lateral rotation of the arm e. Depression of the arm

2. A victim of an automobile accident is unable to abduct her left arm. This indicates damage to which of the following parts of the brachial plexus?

Upper trunk and posterior cord

Popliteus

When standing upright, the femur moves into the locked position by slightly hyperextending, gliding posteriorly, and medially rotating on the tibial plateaus. Which of the following muscles acts to initiate the unlocking of the knee? A. Biceps femoris B. Gastrocnemius C. Popliteus D. Sartorius E. Plantaris

Adductor longus

While inserting a sling through the obturator foramen of an obese 68-year-old woman to treat "coughing urinary incontinence" the surgeon inadvertently transected the obturator nerve at that location. Which of the following muscles would be paralyzed as a result? A. Adductor longus B. Rectus femoris C. Gluteus maximus D. Sartorius E. Biceps femoris

Injury to Axillary Vein

Wounds in the axilla often involve the axillary vein because of its large size and exposed position. When the arm is fully abducted, the axillary vein overlaps the axillary artery anteriorly. A wound in the proximal part of the vein is particularly dangerous, not only because of profuse bleeding but also because of the risk of air entering the vein and producing air emboli (air bubbles) in the blood.

e

10. A 35-year-old man walks in with a stab wound to the most medial side of the proximal portion of the cubital fossa. Which of the following structures would most likely be damaged? (A) Biceps brachii tendon (B) Radial nerve (C) Brachial artery (D) Radial recurrent artery (E) Median nerve

c

1. A 21-year-old patient has a lesion of the upper trunk of the brachial plexus (Erb- Duchenne paralysis). Which of the following is the most likely diagnosis? (A) Paralysis of the rhomboid major (B) Inability to elevate the arm above the horizontal (C) Arm tending to lie in medial rotation (D) Loss of sensation on the medial side of the arm (E) Inability to adduct the thumb

d

10. The given anteroposterior (AP) X-ray depicts a humeral shaft fracture in a 22-year-old man. Given the location of the fracture, which of the following structures is most likely damaged? (A) Posterior circumflex humeral artery (B) Ulnar nerve (C) Axillary nerve (D) Profunda brachii artery (E) Median nerve

c

26. A patient with a deep stab wound in the middle of the forearm has impaired movement of the thumb. Examination indicates a lesion of the anterior interosseous nerve. Which of the following muscles is paralyzed? (A) Flexor pollicis longus and brevis (B) Flexor pollicis longus and opponens pollicis (C) Flexor digitorum profundus and pronator quadratus (D) Flexor digitorum profundus and superficialis (E) Flexor pollicis brevis and pronator quadratus

e

26. Lateral rotation of the arm is an important mechanical component of bringing the arm back when preparing to throw an object. What muscle acts to produce lateral rotation of the arm? (A) Supraspinatus (B) Teres major (C) Latissimus dorsi (D) Subscapularis (E) Teres minor

b

28. Pronator teres syndrome is a condition in which one of the following nerves is excessively compressed where it passes between the two heads of the pronator teres muscle. Which of the following nerves is entrapped? (A) Deep branch of radial nerve (B) Median Nerve (C) Deep branch of ulnar nerve (D) Superficial branch of ulnar nerve (E) Musculocutaneous nerve

c

27. A 29-year-old patient comes in; he cannot flex the distal interphalangeal (DIP) joint of the index finger. His physician determines that he has nerve damage from a supracondylar fracture. Which of the following conditions is also a symptom of this nerve damage? (A) Inability to flex the DIP joint of the ring finger (B) Atrophy of the hypothenar eminence (C) Loss of sensation over the distal part of the second digit (D) Paralysis of all the thumb muscles (E) Loss of supination

b

27. The lateral cord of the brachial plexus is named because it lies immediately lateral to which of the following structures? (A) Long head of the biceps brachii muscle (B) Axillary artery (C) Subclavian vein (D) Surgical neck of the humerus (E) Pectoralis minor muscle

A

1. Physical examination of a 40-year-old man injured in an automobile accident indicates that he has suffered nerve damage affecting his left upper limb. The patient exhibits significant weakness when pronating his left forearm and flexing his left wrist. What nerve is most likely damaged? (A) Median nerve (B) Ulnar nerve (C) Superficial branch of the radial nerve (D) Deep branch of the radial nerve (E) Musculocutaneous nerve

c

28. A 27-year-old man with cubital tunnel syndrome complains of numbness and tingling in the ring and little finger and back and sides of his hand because of damage to a nerve in the tunnel at the elbow. Which of the following muscles is most likely to be paralyzed? (A) Flexor digitorum superficialis (B) Opponens pollicis (C) Two medial lumbricals (D) Pronator teres (E) Supinator

Teres major

A patient has a torn rotator cuff of the shoulder joint as the result of an automobile accident. Which of the following muscle tendons is intact and has normal function? A. Teres major B. Supraspinatus C. Subscapularis D. Teres minor E. Infraspinatus

c

11. A 23-year-old man was injured in a motor vehicle accident and X-rays confirmed a displaced distal radius fracture in his left forearm. Upon examination, the patient exhibits weakened pronation, weakened flexion of the index and middle fingers at the distal interphalangeal joints, and weakened flexion of the interphalangeal joint of the thumb. When asked to make the okay sign (make a circle with the thumb and index finger), the patient is unable to make a round circle, producing a col- lapsed circle on the affected hand (see photo). No areas of sen- sory loss are detected. Which nerve is most likely damaged? (A) Deep branch of the radial nerve (B) Superficial branch of the radial nerve (C) Anterior interosseous nerve (D) Median nerve, proximal to the carpal tunnel (E) Recurrent branch of the median nerve

d

11. The police bring in a murder suspect who has been in a gunfight with a police officer. The suspect was struck by a bullet in the arm; his median nerve has been damaged. Which of the following symptoms is likely produced by this nerve damage? (A) Waiter's tip hand (B) Claw hand (C) Wrist drop (D) Ape hand (E) Flattening of the hypothenar eminence

d

12. A 23-year-old medical student complains of loss of sensation in the skin on the medial edge of her left hand, including the entire fifth digit. The associated motor deficit probably involves weakness in which of the following? (A) Pronation (B) Abduction of the wrist (C) Extension of the wrist (D) Abduction of the index finger (E) Flexion of the interphalangeal joints of the index finger

c

12. An automobile body shop worker has his middle finger crushed while working on a transmission. Which of the following muscles is most likely to retain function? (A) Extensor digitorum (B) Flexor digitorum profundus (C) Palmar interosseous (D) Dorsal interosseous (E) Lumbrical

d

13. A 14-year-old boy falls on his outstretched hand and has a fracture of the scaphoid bone. The fracture is most likely accompanied by a rupture of which of the following arteries? (A) Brachial artery (B) Ulnar artery (C) Deep palmar arterial arch (D) Radial artery (E) Princeps pollicis artery

a

13. A dermatologist performed a biopsy on a suspicious mole on the right side of the posterior neck of a 57-year-old male construction worker. Pathology confirmed a malignant melanoma, so the physician excised a substantial amount of tissue surrounding the mole. After the procedure, the patient experienced difficulty elevating his right shoulder and lifting his right arm over his head. No sensory deficits were seen. What nerve was most likely damaged in this patient? (A) Accessory nerve (B) Axillary nerve (C) Dorsal scapular nerve (D) Long thoracic nerve (E) Thoracodorsal nerve

c

14. A 12-year-old boy walks in; he fell out of a tree and fractured the upper portion of his humerus. Which of the following nerves are intimately related to the humerus and are most likely to be injured by such a fracture? (A) Axillary and musculocutaneous (B) Radial and ulnar (C) Radial and axillary (D) Median and musculocutaneous (E) Median and ulnar

e

15. A 36-year-old man broke a window with his fist to rescue his child from a house fire. The man sustained a laceration to the lateral aspect of his right forearm, but he only showed a sensory deficit (numbness and paresthesia) to the dorsolateral aspect of his hand (as denoted by the shaded area within the given photo). What nerve was most likely damaged? (A) Dorsal cutaneous branch of the ulnar nerve (B) Lateral cutaneous nerve of the forearm (C) Posterior cutaneous nerve of the forearm (D) Deep branch of the radial nerve (E) Superficial branch of the radial nerve

d

15. A man injures his wrist on broken glass. Which of the following structures entering the palm superficial to the flexor retinaculum may be damaged? (A) Ulnar nerve and median nerve (B) Median nerve and flexor digitorum profundus (C) Median nerve and flexor pollicis longus (D) Ulnar artery and ulnar nerve (E) Ulnar nerve and flexor digitorum superficialis

c

16. A patient with Bennett's fracture (a fracture of the base of the first metacarpal bone) experiences an impaired thumb movement. Which of the following intrinsic muscles of the thumb is most likely injured? (A) Abductor pollicis brevis (B) Flexor pollicis brevis (superficial head) (C) Opponens pollicis (D) Adductor pollicis (E) Flexor pollicis brevis (deep head)

d

16. An anesthesiologist administers an anesthetic solution into the axillary sheath of a 19-year-old college baseball player in preparation for repair of the ulnar collateral ligament of the elbow. After 5 minutes, the patient experiences numbness and paresthesia distal to the middle aspect of the arm; however, the medial aspect of the arm and elbow remain sensitive to pain. What nerve provides sensory innervation to the sensitive area and was not blocked by the anesthetic solution? (A) Long thoracic nerve (B) Median nerve (C) Medial cutaneous nerve of the arm (D) Intercostobrachial nerve (E) Ulnar nerve

a

17. A 17-year-old male football player suffers a shoulder injury and arrives at the ER 2 hours after the injury. The physician diagnoses a shoulder dislocation, and after administration of a local anesthetic solution, the doctor repositions the head of the humerus into the glenoid cavity of the scapula (reduction). No fractures are seen on X-rays. However, the patient displays weakness in abduction and external rotation at the shoulder. A loss of sensation is also noted at the superior and lateral aspects of the arm. What nerve was most likely damaged in this injury? (A) Axillary nerve (B) Median nerve (C) Ulnar nerve (D) Radial nerve (E) Musculocutaneous nerve

c

17. A 27-year-old pianist with a known carpal tunnel syndrome experiences difficulty in finger movements. Which of the following intrinsic muscles of her hand is paralyzed? (A) Palmar interossei and adductor pollicis (B) Dorsal interossei and lateral two lumbricals (C) Lateral two lumbricals and opponens pollicis (D) Abductor pollicis brevis and palmar interossei (E) Medial two and lateral two lumbricals

c

18. A 31-year-old roofer walks in with tenosynovitis resulting from a deep penetrated wound in the palm by a big nail. Examination indicates that he has an infection in the ulnar bursa. This infection most likely resulted in necrosis of which of the following tendons? (A) Tendon of the flexor carpi ulnaris (B) Tendon of the flexor pollicis longus (C) Tendon of the flexor digitorum profundus (D) Tendon of the flexor carpi radialis (E) Tendon of the palmaris longus

e

18. A physician tests the myotatic biceps reflex as shown. A normal response of involuntary contraction of the biceps brachii muscle is noted. This reflex confirms the integrity of what nerve? (A) Axillary nerve (B) Median nerve (C) Ulnar nerve (D) Radial nerve (E) Musculocutaneous nerve

d

19. A 21-year-old male college student reports to the student health clinic on Monday morning, the day after the Super Bowl. He explains that he was intoxicated and lost consciousness with his upper limbs draped over the back of a couch. He complains of numbness and paresthesia over the dorsum of his hand on the radial side and is unable to support the weight of his left hand when the hand is placed in a pronated position (see photo). What nerve was most likely damaged in this individual? (A) Axillary nerve (B) Median nerve (C) Ulnar nerve (D) Radial nerve (E) Musculocutaneous nerve

a

19. An 18-year-old boy involved in an automobile accident presents with arm that cannot abduct. His paralysis is caused by damage to which of the following nerves? (A) Suprascapular and axillary (B) Thoracodorsal and upper subscapular (C) Axillary and musculocutaneous (D) Radial and lower subscapular (E) Suprascapular and dorsal scapular

a

2. A 50-year old man falls off a ladder while cleaning his windows, landing on the ground as seen in the given drawing. He does not seek medical aid, believing his general soreness will go away with time. However, after several months, he develops a postural deformity of his left upper limb that includes an adducted, medially rotated, and extended shoulder, extended elbow, and pronated forearm. The injury and subsequent condition reflect damage to what structure? (A) Upper trunk of the brachial plexus (B) Lower roots of the brachial plexus (C) Posterior divisions of the brachial plexus (D) Medial cord of the brachial plexus (E) Lateral root of the median nerve

e

2. A patient comes in with a gunshot wound and requires surgery in which his thoracoacromial trunk needs to be ligated. Which of the following arterial branches would maintain normal blood flow? (A) Acromial (B) Pectoral (C) Clavicular (D) Deltoid (E) Superior thoracic

d

25. A 31-year-old patient complains of sensory loss over the anterior and posterior surfaces of the medial third of the hand and the medial one and one-half fingers. He is diagnosed by a physician as having funny bone symptoms. Which of the following nerves is injured? (A) Axillary (B) Radial (C) Median (D) Ulnar (E) Musculocutaneous

e

25. The pectoralis minor muscle is an important landmark in identifying and describing neighboring structures in the chest and axillary regions. Which of the following relationships of the pectoralis minor is correct? (A) The lateral cord of the brachial plexus lies lateral to the muscle (B) The clavipectoral triangle lies lateral to the muscle (C) The anterior axillary lymph nodes lie along the medial border of the muscle (D) The lateral wall of the axillary fossa includes the muscle (E) The second part of the axillary artery lies deep to the muscle

c

20. A 17-year-old boy with a stab wound received multiple injuries on the upper part of the arm and required surgery. If the brachial artery were ligated at its origin, which of the following arteries would supply blood to the profunda brachii artery? (A) Lateral thoracic (B) Subscapular (C) Posterior humeral circumflex (D) Superior ulnar collateral (E) Radial recurrent

b

20. A 50-year female equestrian is thrown from a startled horse and dragged by the reins, which were wrapped around her left wrist, for some distance. At the ER, she is experiencing pain and paresthesia in the axilla and medial aspect of her upper limb. Despite being left-handed, she has marked weakness in the movements of her dominant hand, especially abduction and adduction of the fingers. What structure was most likely damaged in this woman? (A) Upper trunk of the brachial plexus (B) Lower trunk of the brachial plexus (C) Posterior cord of the brachial plexus (D) Lateral cord of the brachial plexus (E) Long thoracic nerve

d

21. A 23-year-old woman who receives a deep cut to her ring finger by a kitchen knife is unable to move the metacarpophalangeal joint. Which of the following pairs of nerves was damaged? (A) Median and ulnar (B) Radial and median (C) Musculocutaneous and ulnar (D) Ulnar and radial (E) Radial and axillary

d

21. A child is born to a young woman who had utilized thalidomide to help relieve her morning sickness early in her pregnancy. The infant suffers the congenital defects shown here. She is missing the proximal segments of both upper and lower limbs. The hands and feet that are present are attached to the trunk of the body and resemble small seal's flippers. Which of the following is the correct term for this malformation? (A) Adysplasia (B) A duplication defect (C) Micromelia (D) Meromelia (E) Amelia

d

22. A 23-year-old competitive weight lifter goes to his physician complaining of pain in his proximal forearm. During his examination, the pain is exacerbated by flexion of the elbow and supination of the forearm against resistance. A lateral radiograph shows chronic microtrauma to the proximal radius, marked by the black arrows. Which of the following muscles attaches to, and most likely damaged, this osteological process? (A) Supinator (B) Brachialis (C) Pronator teres (D) Biceps brachii (E) Brachioradialis

b

22. A 27-year-old baseball player is hit on his forearm by a high-speed ball during the World Series, and the muscles that form the floor of the cubital fossa appear to be torn. Which of the following groups of muscles have lost their functions? (A) Brachioradialis and supinator (B) Brachialis and supinator (C) Pronator teres and supinator (D) Supinator and pronator quadratus (E) Brachialis and pronator teres

b

23. A 23-year-old man complains of numbness on the medial side of the arm following a stab wound in the axilla. On examination, he is diagnosed with an injury of his medial brachial cutaneous nerve. In which of the following structures are the cell bodies of the damaged nerve involved in numbness located? (A) Sympathetic chain ganglion (B) Dorsal root ganglion (C) Anterior horn of the spinal cord (D) Lateral horn of the spinal cord (E) Posterior horn of the spinal cord

b

23. A 65-year-old man is brought to the emergency room after being attacked in his office by a disgruntled co-worker. The attacker reportedly used a long, narrow-bladed letter-opener to inflict multiple stab wounds to the man's back. Physical examination shows a puncture wound in the posterior axillary fold. The patient presents with weakness in extension, adduction, and medial rotation of his arm. Which of the following muscles is most likely cut in this injury? (A) Pectoralis minor (B) Latissimus dorsi (C) Levator scapulae (D) Serratus anterior (E) Teres minor

d

24. A 38-year-old homebuilder was involved in an accident and is unable to supinate his forearm. Which of the following nerves are most likely damaged? (A) Suprascapular and axillary (B) Musculocutaneous and median (C) Axillary and radial (D) Radial and musculocutaneous (E) Median and ulnar

d

24. Physical examination of a 45-year-old man who had been stabbed in the back of the shoulder shows a deep wound penetrating into the quadrangular space of the shoulder, causing bleeding from the severed blood vessels there. Which of the following neural structures is most likely damaged as well? (A) Musculocutaneous nerve (B) Lateral cord of the brachial plexus (C) Radial nerve (D) Axillary nerve (E) Medial cutaneous nerve of the arm

c

29. A secretary comes in to your office complaining of pain in her wrists from typing all day. You determine that she likely has carpal tunnel syndrome. Which of the following conditions would help you determine the diagnosis? (A) Inability to adduct the little finger (B) Inability to flex the DIP joint of the ring finger (C) Flattened thenar eminence (D) Loss of skin sensation of the medial one and one-half fingers (E) Atrophied adductor pollicis muscle

c

29. The pulse of the radial artery is readily palpable where the vessel passes which of the following structures? (A) Across the anterior aspect of the lateral epicondyle of the humerus (B) Between the tendons of the palmaris longus and flexor carpi ulnaris (C) Lateral to the tendon of the flexor carpi radialis (D) Superficial to the tendons of the extensor pollicis brevis and abductor pollicis longus (E) Superficial to the carpal tunnel

B

3. A 21-year-old man goes to his college campus health clinic complaining of soreness in his left wrist after falling on an outstretched hand during a basketball game the previous day. He is supporting his left wrist and indicates that the pain worsens with movement and is minimized with inactivity. There is no loss of feeling in his hand, nor does he have trouble grasping or holding objects. The physician exacerbates the wrist pain by applying pressure to the base of the thumb in the anatomical snuffbox (see photo). Radiographic imaging will confirm a break of which carpal bone? (A) Capitate (B) Scaphoid (C) Hamate (D) Trapezium (E) Pisiform

d

3. A 29-year-old man comes in with a stab wound, cannot raise his arm above horizontal, and exhibits a condition known as winged scapula. Which of the following structures of the brachial plexus would most likely be damaged? (A) Medial cord (B) Posterior cord (C) Lower trunk (D) Roots (E) Upper trunk

b

45. A 48-year-old woman is diagnosed with a malignant tumor in the superolateral quadrant of the right breast, including the axillary tail. If it metastasizes, this cancer will most likely spread first to which of the following locations? (A) Lateral axillary lymph nodes (B) Anterior axillary lymph nodes (C) Deep cervical lymph nodes (D) Parasternal lymph nodes (E) Contralateral breast lymph nodes

e

33. During an attempted suicide, a depressed young woman slashes the front of her wrist with a razor blade. However, she cuts only to the depth of the superficial aspect of the flexor retinaculum before passing out at the sight of her own blood. Which of the following muscle tendons may be severed? (A) Flexor digitorum superficialis (B) Brachioradialis (C) Flexor pollicis longus (D) Abductor pollicis longus (E) Flexor carpi radialis

d

33. A patient complains of having pain with repeated movements of his thumb (claudication). His physician performs the Allen test and finds an insufficiency of the radial artery. Which of the following conditions would be a result of the radial artery stenosis? (A) A marked decrease in the blood flow in the superficial palmar arterial arch (B) Decreased pulsation in the artery passing superficial to the flexor retinaculum (C) Ischemia of the entire extensor muscles of the forearm (D) A marked decrease in the blood flow in the princeps pollicis artery (E) A low blood pressure in the anterior interosseous artery

d

30. A 74-year-old man complains of pain in his right hand and fingers when he works with his hands for a while. Thorough testing reveals insufficient blood flow into the deep palmar arch. Occlusion of which of the following arteries is the most likely cause of this condition? (A) Posterior interosseous artery (B) Ulnar artery (C) Anterior interosseous artery (D) Radial artery (E) Inferior ulnar collateral artery

c

30. A man is unable to hold typing paper between his index and middle fingers. Which of the following nerves was likely injured? (A) Radial nerve (B) Median nerve (C) Ulnar nerve (D) Musculocutaneous nerve (E) Axillary nerve

e

31. In both the upper and lower limbs, the superficial veins begin in a dorsal cutaneous arch that drains into medial and lateral cutaneous veins aligned mainly along the first and fifth digit sides of the limb. Which of the following veins in the upper limb is the equivalent of the great saphenous vein in the lower limb? (A) Radial vein (B) Ulnar vein (C) Brachial vein (D) Basilic vein (E) Cephalic vein

d

31. The victim of an automobile accident has a destructive injury of the proximal row of carpal bones. Which of the following bones is most likely damaged? (A) Capitate (B) Hamate (C) Trapezium (D) Triquetrum (E) Trapezoid

c

32. A patient has a torn rotator cuff of the shoulder joint as the result of an automobile accident. Which of the following muscle tendons is intact and has normal function? (A) Supraspinatus (B) Subscapularis (C) Teres major (D) Teres minor (E) Infraspinatus

a

32. The given X-ray reveals a fracture of the proximal humerus, indicated by the black arrow. Given the location of the fracture, what artery is most likely damaged in this patient? (A) Posterior circumflex humeral artery (B) Brachial artery (C) Deep brachial (profunda brachii) artery (D) Subscapul arartery (E) Superior ulnar collateral artery

e

34. A 17-year-old man has pain and moderate swelling over the dorsomedial aspect and in the hypothenar area of his right hand after punching a locker over a dispute with his girlfriend. What is the most likely finding on an X-ray of his hand? (A) Dislocation of the fifth metacarpophalangeal joint (B) Fracture of the triquetral bone (C) Fracture of the proximal phalanx of the ring finger (D) Fracture of the proximal phalanx of the little finger (E) Fracture of the fifth metacarpal bone

a

34. A patient bleeding from the shoulder secondary to a knife wound is in fair condition because there is vascular anastomosis around the shoulder. Which of the following arteries is most likely a direct branch of the subclavian artery that is involved in the anastomosis? (A) Dorsal scapular artery (B) Thoracoacromial artery (C) Circumflex scapular artery (D) Transverse cervical artery (E) Suprascapular artery

b

35. A right-handed 21-year-old college student visits his physician because of pain in his right shoulder that developed after starting a summer job on a construction crew 2 weeks ago. He explains that on his job site he regularly lifts heavy construction materials over his head. During physical examination, the patient experiences sharp pain in the range of 80 to 150 degrees of abduction at the glenohumeral joint. What is the most likely diagnosis? (A) Infraspinatus tendonitis (B) Supraspinatus tendonitis (C) Acromioclavicular (AC) joint arthritis (D) Degenerative arthritis of the shoulder (E) Broken clavicle

d

35. During a breast examination of a 56-year-old woman, the physician found a palpable mass in her breast. Which of the following characteristics of breast cancer and its diagnosis is correct? (A) Elevated nipple (B) Polymastia (C) Shortening of the clavipectoral fascia (D) Dimpling of the overlying skin (E) Enlargement of the breast

a

36. A 56-year-old woman was stopped at a light when her car was rear-ended by another car. She had her right arm on the steering wheel, and the impact caused forced flexion at her elbow. Several months later, she comes to her physician complaining of numbness and a pins and needles sensation in her right little finger when she talks on the phone, rests her head on her right hand at work, or spends most of her day typing at work. She also notices the quality of her typing and her ability to play the violin have diminished. Which nerve is compressed at what location? (A) Ulnar nerve in the elbow (B) Ulnar nerve in the wrist (C) Median nerve in the wrist (D) Median nerve in the elbow (E) Median nerve in the axilla

a

36. A patient with a stab wound receives a laceration of the musculocutaneous nerve. Which of the following conditions is most likely to have occurred? (A) Lack of sweating on the lateral side of the forearm (B) Inability to extend the forearm (C) Paralysis of brachioradialis muscle (D) Loss of tactile sensation on the arm (E) Constriction of blood vessels on the hand

e

37. A 20-year-old man fell from the parallel bar during the Olympic trial. A neurologic examination reveals that he has a lesion of the lateral cord of the brachial plexus. Which of the following muscles is most likely weakened by this injury? (A) Subscapularis (B) Teres major (C) Latissimus dorsi (D) Teres minor (E) Pectoralis major

e

37. As part of a physical examination to evaluate intrinsic hand muscle function, a physician holds three fingers in the extended position, and instructs the patient to flex the proximal interphalangeal joint of the free finger, as shown. Which of the following muscles is the doctor specifically testing? (A) Flexor digitorum profundus (FDP) (B) Extensor digitorum (C) Second lumbrical (D) Dorsal interosseous (E) Flexor digitorum superficialis (FDS)

d

38. A 24-year-old carpenter suffers a crush injury of his entire little finger. Which of the following muscles is most likely to be spared? (A) Flexor digitorum profundus (B) Extensor digitorum (C) Palmar interossei (D) Dorsal interossei (E) Lumbricals

c

38. Because of repeated bad needle sticks, a heroin addict develops an infected abscess in the floor of the cubital fossa. Which of the following structures is the abscess most likely to invade first? (A) Brachioradialis muscle (B) Pronator teres muscle (C) Brachialis muscle (D) Head of the radius (E) Olecranon fossa of the humerus

d

39. A 7-year-old boy falls from a tree house and is brought to the emergency department of a local hospital. On examination, he has weakness in rotating his arm laterally because of an injury of a nerve. Which of the following conditions is most likely to cause a loss of this nerve function? (A) Injury to the lateral cord of the brachial plexus (B) Fracture of the anatomic neck of the humerus (C) Knife wound on the teres major muscle (D) Inferior dislocation of the head of the humerus (E) A tumor in the triangular space in the shoulder region

a

39. The traditional radical mastectomy includes removal of the pectoralis major muscle. Which of the following movements is most affected postoperatively by this surgical procedure? (A) Adduction of the arm (B) Abduction of the arm (C) Extension of the arm (D) Lateral rotation of the arm (E) Depression of the arm

e

4. A 16-year-old patient has weakness flexing the metacarpophalangeal joint of the ring finger and is unable to adduct the same finger. Which of the following muscles is most likely paralyzed? (A) Flexor digitorum profundus (B) Extensor digitorum (C) Lumbrical (D) Dorsal interosseous (E) Palmar interosseous

a

4. As a result of chronic stress associated with an intense high school weight-lifting program, a 15-year-old boy suffers an avulsion fracture of the greater tubercle of the humerus. In the ER, he displays difficulty initiating abduction of the upper limb. Which of the following muscles was involved in this fracture? (A) Supraspinatus (B) Long head of biceps brachii (C) Long head of triceps (D) Subscapularis (E) Infraspinatus

d

43. As part of a physical examination to evaluate intrinsic hand muscle function, a physician asks the patient to assume the Z-position (seen in photo) with his hand, which involves flexion of the metacarpophalangeal joints and extension of the interphalangeal joints of the fingers. Which of the following nerves is being tested in assuming this position? (A) Deep branch of radial nerve (B) Superficial branch of radial nerve (C) Recurrent branch of median nerve (D) Deep branch of ulnar nerve (E) Superficial branch of ulnar nerve

e

45. The lateral portion of the fractured clavicle is displaced downward by which of the following? (A) Deltoid and trapezius muscles (B) Pectoralis major and deltoid muscles (C) Pectoralis minor muscle and gravity (D) Trapezius and pectoralis minor muscles (E) Deltoid muscle and gravity

c

44. A 10-year-old boy was running across a parking lot when he tripped and received lacerations on the base of his thumb from a broken glass bottle. On examination, his thumb was unable to oppose to his fingers, and the thumb also showed weakness when abducting and flexing. No sensory deficits were reported. What nerve was most likely severed? (A) Deep branch of radial nerve (B) Superficial branch of radial nerve (C) Recurrent branch of median nerve (D) Deep branch of ulnar nerve (E) Superficial branch of ulnar nerve

c

44. A 22-year-old female Macarena dancer fell from the stage and complains of elbow pain and inability to supinate her forearm. Which of the following nerves are most likely injured from this accident? (A) Median and ulnar nerves (B) Axillary and radial nerves (C) Radial and musculocutaneous nerves (D) Ulnar and axillary nerves (E) Musculocutaneous and median nerves

b

51. Which of the following nerves is most likely injured as a result of this accident? (A) Musculocutaneous (B) Axillary (C) Radial (D) Median (E) Ulnar

b

40. A 49-year-old woman is diagnosed as having a large lump in her right breast. Lymph from the cancerous breast drains primarily into which of the following nodes? (A) Apical nodes (B) Anterior (pectoral) nodes (C) Parasternal (internal thoracic) nodes (D) Supraclavicular nodes (E) Nodes of the anterior abdominal wall

b

40. The lateral thoracic artery provides the main blood supply to the lateral side of the chest wall, including much of the breast. To deter excessive blood loss during a surgical procedure involving the breast, a surgeon can clamp the lateral thoracic artery near its origin. Which of the following arteries gives rise to this artery? (A) First part of the axillary artery (B) Second part of the axillary artery (C) Third part of the axillary artery (D) Third part of the subclavian artery (E) First part of the brachial artery

c

41. A 17-year-old boy fell from his motorcycle and complains of numbness of the lateral part of the arm. Examination reveals that the axillary nerve is severed. Which of the following types of axons is most likely spared? (A) Postganglionic sympathetic axons (B) Somatic afferent axons (C) Preganglionic sympathetic axons (D) General somatic efferent axons (E) General visceral afferent axons

d

41. Following a radical mastectomy procedure, a surgeon plans to conduct a breast reconstruction utilizing a latissimus dorsi muscle flap. What nerve will the surgeon need to keep intact during the surgical dissection of the chest wall to prevent atrophy of the muscle flap? (A) Long thoracic nerve (B) Intercostobrachial nerve (C) Medial pectoral nerve (D) Thoracodorsal nerve (E) Axillary nerve

e

42. A construction worker suffers a destructive injury of the structures related to the anatomic snuffbox. Which of the following structures would most likely be damaged? (A) Triquetral bone (B) Trapezoid bone (C) Extensor indicis tendon (D) Abductor pollicis brevis tendon (E) Radial artery

b

42. An 80-year-old woman comes to the physician because of a lump in her right breast. Physical examination shows a 2-cm mass in the right breast with dimpling of the overlying skin and peau d'orange (edema of the breast with the skin assuming the appearance of an orange peel). Examination of a biopsy specimen confirms a diagnosis of carcinoma. Involvement of what structure is the most likely cause of this patient's skin dimpling? (A) Clavipectoral fascia (B) Suspensory ligaments (C) Lactiferous ducts (D) Retromammary space (E) Pectoralis major

c

43. A rock climber falls on his shoulder, resulting in a chipping off of the lesser tubercle of the humerus. Which of the following structures would most likely have structural and functional damage? (A) Supraspinatus muscle (B) Infraspinatus muscle (C) Subscapularis muscle (D) Teres minor muscle (E) Coracohumeral ligament

d

46. A 48-year-old woman falls on an icy sidewalk and lands on her right elbow. She suffers a midshaft humeral fracture, as seen on the given X-ray. The attending physician wants to assess whether the nerve residing in the spiral groove of the humerus is damaged. What sign or symptom would confirm damage to this nerve? (A) Numbness on the lateral (radial) aspect of the forearm (B) Numbness of the medial aspect of the upper arm (C) Numbness over the superolateral aspect of the upper arm (D) Weakness extending the wrist and fingers (E) Weakness in grip strength

d

46. Which of the following muscles causes upward displacement of the medial fragment? (A) Pectoralis major (B) Deltoid (C) Trapezius (D) Sternocleidomastoid (E) Scalenus anterior

a

47. Organization of the axillary lymph nodes into Levels I, II, and III for breast cancer treatment is based on the location of the nodes relative to which of the following muscles? (A) Pectoralis minor (B) Pectoralis major (C) Latissimus dorsi (D) Serratus anterior (E) Subscapularis

b

47. Which of the following conditions is most likely to occur secondary to the fractured clavicle? (A) A fatal hemorrhage from the brachiocephalic vein (B) Thrombosis of the subclavian vein, causing a pulmonary embolism (C) Thrombosis of the subclavian artery, causing an embolism in the ascending aorta (D) Damage to the upper trunk of the brachial plexus (E) Damage to the long thoracic nerve, causing the winged scapula

e

48. A 37-year-old factory worker fractures multiple bones distal to the elbow when his hand and forearm are crushed by equipment dropped by a faulty hydraulic lift. Which of the following bones, if fractured, would most likely develop avascular necrosis? (A) Distal radius (B) Midshaft ulna (C) Fifth metacarpal (D) Lunate (E) Scaphoid

e

48. Which of the following nerves is most likely injured as a result of this accident? (A) Axillary (B) Musculocutaneous (C) Radial (D) Median (E) Ulnar

c

49. As part of a physical examination to evaluate muscle function in the hand, a physician holds the four fingers (digits 2 through 5) and asks the patient to spread their fingers, as shown below. What muscle(s) is/are the doctor testing? (A) Lumbrical muscles (B) Palmar interosseous muscles (C) Dorsal interosseous muscles (D) Flexor digitorum superficialis (E) Flexor digitorum profundus

d

49. Which of the following muscles is most likely paralyzed as a result of this accident? (A) Extensor pollicis brevis (B) Abductor pollicis longus (C) Abductor pollicis brevis (D) Adductor pollicis (E) Opponens pollicis

c

5. A 27-year-old patient presents with an inability to draw the scapula forward and downward because of paralysis of the pectoralis minor. Which of the following would most likely be a cause of his condition? (A) Fracture of the clavicle (B) Injury to the posterior cord of the brachial plexus (C) Fracture of the coracoid process (D) Axillary nerve injury (E) Defects in the posterior wall of the axilla

e

5. During an attempted suicide, a depressed young woman slashes her wrist with a straight razor. She cuts just proximal to the pisiform bone to the depth of the superficial aspect of the flexor retinaculum before passing out at the sight of her own blood. As a result of this wound, she may suffer a neuromuscular deficit that results in which of the following? (A) Weakness in pronation (B) Inability to abduct the thumb (C) Weakness in flexion of the thumb (D) Weakness in opposition of the thumb (E) Inability to adduct the thumb

b

50. A 52-year-old retired professional cyclist, who still rides his bike 400 miles per week, comes to his physician complaining of hand problems. The physician notes hyperextension of the ring and little fingers at the metacarpophalangeal joints and flexion at the interphalangeal joints within the same fingers (see photo). During examination, the patient has no weakness in flexion or adduction of the wrist. What nerve is compressed at what location? (A) Ulnar nerve in the elbow (B) Ulnar nerve in the wrist (C) Median nerve in the wrist (D) Median nerve in the elbow (E) Median nerve in the axilla

e

50. After this injury, the patient is unable to do which of the following? (A) Flex the proximal interphalangeal joint of his ring finger (B) Flex the DIP joint of his index finger (C) Feel sensation on his middle finger (D) Abduct his thumb (E) Adduct his index finger

e

51. A 3-year-old girl is brought to the emergency room holding her right arm with the elbow flexed and the forearm pronated. She refuses to move her arm and complains her elbow hurts a lot. Her mother reports they were holding hands and running in the park when the child tripped. The mother pulled on the child's hand to prevent her from hitting the ground. Given the nature of this injury and the age of the patient, what structure is most likely damaged? (A) Interosseous membrane of forearm (B) Quadrate ligament (C) Radial collateral ligament of elbow (D) Ulnar collateral ligament of elbow (E) Anular ligament of radius

c

52. Following this accident, the damaged nerve causes difficulty in abduction, extension, and lateral rotation of his arm. Cell bodies of the injured nerve involved in movement of his arm are located in which of the following structures? (A) Dorsal root ganglion (B) Sympathetic chain ganglion (C) Anterior horn of the spinal cord (D) Lateral horn of the spinal cord (E) Posterior horn of the spinal cord

d

53. The damaged nerve causes numbness of the lateral side of the arm. Cell bodies of the injured nerve fibers involved in sensory loss are located in which of the following structures? (A) Anterior horn of the spinal cord (B) Posterior horn of the spinal cord (C) Lateral horn of the spinal cord (D) Dorsal root ganglia (E) Sympathetic chain ganglia

c

54. This accident most likely leads to damage of which of the following arteries? (A) Axillary (B) Deep brachial (C) Posterior humeral circumflex (D) Superior ulnar collateral (E) Scapular circumflex

b

55. Following this accident, the boy has weakness in rotating his arm laterally. Which of the following muscles are paralyzed? (A) Teres major and teres minor (B) Teres minor and deltoid (C) Infraspinatus and deltoid (D) Supraspinatus and subscapularis (E) Teres minor and infraspinatus

a

56. The blood most likely comes from which of the following arteries? (A) Brachial (B) Radial (C) Ulnar (D) Common interosseous (E) Superior ulnar collateral

b

57. During the procedure, the needle hits a nerve medial to the artery. Which of the following nerves is most likely damaged? (A) Radial (B) Median (C) Ulnar (D) Lateral antebrachial (E) Medial antebrachial

b

58. Which of the following nerves is most likely damaged? (A) Axillary nerve (B) Radial nerve (C) Musculocutaneous nerve (D) Median nerve (E) Ulnar nerve

d

59. As a result of this fracture, the patient shows lack of sweating on the back of the arm and forearm. Cell bodies of the damaged nerve fibers involved in sweating are located in which of the following structures? (A) Anterior horn of the spinal cord (B) Posterior horn of the spinal cord (C) Lateral horn of the spinal cord (D) Sympathetic chain ganglion (E) Dorsal root ganglion

a

6. As part of a physical examination to evaluate muscle function in the hand, a physician holds the proximal interphalangeal joint of his patient's index finger in the extended position and instructs him to try to flex the distal interphalangeal joint, as shown below. Which of the following muscles is the doctor testing? (A) Flexor digitorum profundus (FDP) (B) Extensor indicis (C) First lumbrical (D) First dorsal interosseous (E) Flexor digitorum superficialis (FDS)

d

60. Following this accident, the patient has no cutaneous sensation in which of the following areas? (A) Medial aspect of the arm (B) Lateral aspect of the forearm (C) Palmar aspect of the second and third digits (D) Area of the anatomic snuffbox (E) Medial one and one-half fingers

c

61. Which of the following arteries may be damaged? (A) Brachial artery (B) Posterior humeral circumflex artery (C) Profunda brachii artery (D) Radial artery (E) Radial recurrent artery

d

62. After this accident, supination is still possible through contraction of which of the following muscles? (A) Supinator (B) Pronator teres (C) Brachioradialis (D) Biceps brachii (E) Supraspinatus

d

63. If the structure indicated by the letter A is calcified, which of the following muscles is most likely paralyzed? (A) Deltoid (B) Teres major (C) Teres minor (D) Infraspinatus (E) Subscapularis

d

64. If the structure indicated by the letter B is fractured, which of the following structures is most likely injured? (A) Musculocutaneous nerve (B) Radial nerve (C) Deep brachial artery (D) Posterior humeral circumflex artery (E) Scapular circumflex artery

b

65. Destruction of this area would most likely cause weakness of supination and flexion of the forearm.

e

66. Destruction of this area would most likely cause weakness of pronation of the forearm and flexion of the wrist joints.

d

67. A lesion of the radial nerve would most likely cause paralysis of muscles that are attached to this area.

c

68. Destruction of the structure indicated by the letter E most likely causes weakness of which of the following muscles? (A) Flexor carpi radialis (B) Palmaris longus (C) Flexor carpi ulnaris (D) Brachioradialis (E) Flexor digitorum superficialis

b

69. If the floor of the anatomic snuffbox and origin of the abductor pollicis brevis are damaged, which of the following bones is most likely to be involved? (A) A (B) B (C) C (D) D (E) E

a

7. A 16 year-old boy fell from a motorcycle, and his radial nerve was severely damaged because of a fracture of the midshaft of the humerus. Which of the following conditions would most likely result from this accident? (A) Loss of wrist extension leading to wrist drop (B) Weakness in pronating the forearm (C) Sensory loss over the ventral aspect of the base of the thumb (D) Inability to oppose the thumb (E) Inability to abduct the fingers

b

7. A 17-year-old boy comes to the emergency room after a hard fall onto the lateral aspect of his left shoulder during a high school basketball game. He complains of generalized pain during shoulder motion. On physical examination, the distal end of the clavicle is prominent and distinctly palpable. Radiological findings confirm the diagnosis of a severe (grade 3) shoulder separation. Which of the following features is a component of this condition? (A) Dislocated head of the humerus (B) Torn coracoclavicular ligament (C) Fractured clavicle (D) Dislocated sternal end of the clavicle (E) Torn anterior glenohumeral (GH) ligament

d

70. The patient is unable to abduct her middle finger because of paralysis of this structure.

a

71. A lesion of the median nerve causes paralysis of this structure.

g

72. The patient is unable to adduct her little finger because of paralysis of this structure.

e

73. Atrophy of this structure impairs extension of both the metacarpophalangeal and interphalangeal joints.

d

8. A patient comes in complaining that she cannot flex her proximal interphalangeal joints. Which of the following muscles appear(s) to be paralyzed on further examination of her finger? (A) Palmar interossei (B) Dorsal interossei (C) Flexor digitorum profundus (D) Flexor digitorum superficialis (E) Lumbricals

c

8. An 18-year-old boy is cut severely on the lateral wall of his right chest during a knife fight. Following healing, his scapula moves away from the thoracic wall when he leans on his right hand, giving the appearance indicated in the given photo. Which of the following nerves is likely damaged? (A) Axillary nerve (B) Thoracodorsal nerve (C) Long thoracic nerve (D) Dorsal scapular nerve (E) Suprascapular nerve

c

9. A 21-year-old woman walks in with a shoulder and arm injury after falling during horseback riding. Examination indicates that she cannot adduct her arm because of paralysis of which of the following muscles? (A) Teres minor (B) Supraspinatus (C) Latissimus dorsi (D) Infraspinatus (E) Serratus anterior

b

9. On his downswing, an amateur golfer strikes the hard earth with his club and feels pain in his right wrist. During a subsequent physical examination, he complains of wrist pain that is exacerbated by gripping, displays point tenderness in his medial wrist, and complains of numbness and weakness in his pinky finger (fifth digit). What carpal bone, identified by the white arrow on the given X-ray, is most likely fractured in this patient? (A) Capitate (B) Hamate (C) Lunate (D) Pisiform (E) Scaphoid

Axillary

A 10-year-old boy falls off his bike, has difficulty in moving his shoulder, and is brought to an emergency department. His radiogram and angiogram reveal fracture of the surgical neck of his humerus and bleeding from the point of the fracture. Which of the following nerves is most likely injured as a result of this accident? a. Axillary b. Musculocutaneous c. Radial d. Median e. Ulnar

Posterior circumflex humeral artery

A 17-year-old boy is taken to the emergency room after being struck on the lateral aspect of the left shoulder by a baseball bat during a gang fight. Examination reveals a fracture of the surgical neck of the left humerus, displaced bone fragments piercing the quadrangular space of the shoulder, and deep blood accumulation around the fracture. Which of the following blood vessels was most likely severed? A. Cephalic vein B. Circumflex scapular artery C. Posterior circumflex humeral artery D. Profunda brachii artery E. Subscapular artery

Radial nerve

A 21-year-old male college student reports to the student health clinic on Monday morning, the day after the Super Bowl. He explains that he was intoxicated and lost consciousness with his upper limbs draped over the back of a couch. He complains of numbness and paresthesia over the dorsum of his hand on the radial side and is unable to support the weight of his left hand when the hand is placed in a pronated position (see photo). What nerve was most likely damaged in this individual? A. Axillary nerve B. Median nerve C. Ulnar nerve D. Radial nerve E. Musculocutaneous nerve

Rhomboid major

A 25-year-old man with congenital abnormalities at birth has a lesion of the dorsal scapular nerve, making him unable to adduct his scapula. Which of the following muscles is most likely paralyzed? A. Rhomboid major B. Semispinalis capitis C. Multifidus D. Rotator longus E. Iliocostalis

Rotation

A 26-year-old heavyweight boxer was punched on his mandible, resulting in a slight subluxation (dislocation) of the atlantoaxial joint. The consequence of the injury was decreased range of motion at that joint. What movement would be most affected? A. Rotation B. Extension C. Flexion D. Abduction E. Adduction

Ulnar

A 31-year-old patient complains of sensory loss over the anterior and posterior surfaces of the medial third of the hand and the medial one and one-half fingers. He is diagnosed by a physician as having "funny bone" symptoms. Which of the following nerves is injured? A. Ulnar B. Axillary C. Radial D. Median E. Musculocutaneous

Rupture of the tendon of the long head of the biceps

A 33-year-old tennis player initially experienced vague pain in the anterior region of her glenohumeral joint during a game. Then, she had the sensation of something popping during her third set. During the preliminary examination by her trainer, he noted tenderness over the intertubercular groove in the humerus, flexion and supination weakness of the forearm, and an abnormal bulge in the distal part of the anterior part of her arm. Radiographs of the shoulder and arm did not reveal skeletal abnormalities. Which of the following statements best describes the most likely cause of the symptoms and signs in this case? A. Rupture of the tendon of the long head of the biceps B. Avulsion of the supraglenoid tubercle of the scapula C. Rupture of the tendon of the short head of the biceps D. Rupture of the long head of the triceps brachii tendon E. Rupture of the subscapularis tendon

Supraspinatus

A 35-year-old left-handed baseball pitcher experiences a severe spasm of pain while abducting his left arm in beginning to throw a pitch, and is unable to continue playing. Subsequently, he cannot initiate abduction of the left arm. However, if that arm is passively elevated through the first 15° of abduction, he can complete bringing the arm up to a right angle. The tendon of which of the following muscles is most likely torn? A. Infraspinatus B. Subscapularis C. Supraspinatus D. Teres major E. Teres minor

Median nerve

A 36-year-old plumber slips and breaks a porcelain sink, cutting an anterior aspect of his wrist deeply on a sharp edge. On arrival to the emergency department, he can adduct his thumb but not oppose it. Which of the following structures has been severed? A. Median nerve B. Radial nerve C. Ulnar nerve D. Anterior interosseous nerve E. Posterior interosseous nerve

Scaphoid

A 37-year-old factory worker fractures multiple bones distal to the elbow when his hand and forearm are crushed by equipment dropped by a faulty hydraulic lift. Which of the following bones, if fractured, would most likely develop avascular necrosis? a. Distal radius b. Midshaft ulna c. Fifth metacarpal d. Lunate e. Scaphoid

Radial and musculocutaneous

A 38-year-old homebuilder was involved in an accident and is unable to supinate his forearm. Which of the following nerves are most likely damaged? a. Radial and musculocutaneous b. Suprascapular and axillary c. Musculocutaneous and median d. Axillary and radial e. Median and ulnar

Median nerve

A 42-year-old woman presents to an outpatient clinic with a 6-month history of numbness and tingling on the palmar aspect of her lateral three and one-half fingers, loss of pronation, and flattening of the thenar eminence. Injury to which of the following nerves could cause such a condition? A. Median nerve B. Axillary nerve C. Musculocutaneous nerve D. Radial nerve E. Ulnar nerve

Distal (carpal) compression of the median nerve

A 45-year-old secretary experienced "pins and needles" sensations (paresthesia) and pain in her right hand during the night. These sensations involved the palmar surface of her thumb and the lateral two and a half fingers. Her colleagues told her that she probably has carpal tunnel syndrome. Her family physician confirmed that this was the condition she was experiencing. She also told the physician that she was having some difficulty typing. On further examination, the physician detected some weakness of her right thenar muscles when compared with the left side. Otherwise motor functions were unimpaired currently. Which of the following statements best describes the most likely cause of the paresthesia experienced by the secretary? A. Distal (carpal) compression of the median nerve B. Compression of the ulnar nerve C. Proximal trauma to the median nerve D. Injury of the radial nerve in the arm E. Compression of the radial artery at the wrist

Jefferson (burst) fracture of C1

A 46-year-old supervisor was reading a work order on a construction site when a 60 lb bag of concrete mix was accidentally dropped on the apex of his head. He was immobilized and brought to the ER where he presented with upper neck pain but no neurological signs. Based upon the given axial CT scan and the patient's presentation, which of the following diagnoses is most likely? A. Damage to the cervical spinal cord B. No pathology is apparent on the CT scan C. Jefferson (burst) fracture of C1 D. Fracture of the dens axis (odontoid process) E. Atlanto-axial subluxation

Anterior (pectoral) nodes

A 49-year-old woman is diagnosed as having a large lump in her right breast. Lymph from the cancerous breast drains primarily into which of the following nodes? a. Anterior (pectoral) nodes b. Apical nodes c. Parasternal (internal thoracic) nodes d. Supraclavicular nodes e. Nodes of the anterior abdominal wall

Has torn the long head of his biceps brachii

A 56-year-old avid tennis player is seeing you in the ED after a game in which he heard a popping sound and now has shoulder/arm pain. There is a ball-like swelling near the anterior distal part of his arm consistent with a Popeye deformity. You suspect the patient has which of the following? a. Hemorrhage into his brachialis b. Hemorrhage into his biceps brachii c. Has torn the long head of his biceps brachii d. Has torn the short head of his biceps brachii e. Has torn his coracobrachialis

Upper lateral quadrant of the gluteal region

A 67-year-old patient has been given a course of antibiotics by gluteal intramuscular injections after a major abdominal surgery. To avoid damaging the sciatic nerve during an injection, the needle should be inserted into which of the following areas? A. Upper lateral quadrant of the gluteal region B. Over the sacrospinous ligament C. Midway between the ischial tuberosity and the lesser trochanter D. Midpoint of the gemelli muscles E. Lower medial quadrant of the gluteal region

Brachioradialis

A biomedical engineer would like to reconstruct the arm of a boy who underwent amputation to treat a life-threatening infection. In designing the prosthetic arm, the engineer will need to know that which of the following muscles flexes the elbow and is innervated by the radial nerve? A. Brachioradialis B. Flexor digitorum longus C. Brachialis D. Extensor digitorum longus E. Biceps brachii

Extensor pollicis longus

A college student goes to the student health center complaining that he badly hurt his right hand during an intramural rugby game. Initial examination indicates a fracture dislocation of the distal phalanx of the thumb. Further, the normally distinct medial wall of the anatomical snuff box is not apparent. Which of the following muscle tendons is most likely ruptured in this injury? a. Abductor pollicis brevis b. Extensor carpi radialis longus c. Extensor pollicis longus d. Extensor indicis e.Flexor pollicis longus

Severance of the median nerve

A depressed 14-year-old girl slashed her right wrist with a razor blade. She was rushed to the emergency department. The small spurts of blood from her wrist were difficult to stop. Physical examination of her wrist and hand revealed that a large nerve was severed. She could adduct, abduct, and extend her thumb but was unable to oppose it. She had lost some fine control of the movements of her second and third digits and there was loss of sensation over the lateral half of her palm and digits. Which of the following statements best describes the most likely cause of the signs resulting from severance of the nerve? a. Severance of the median nerve b. Severance of the ulnar nerve c. Severance of the radial nerve d. Severance of the recurrent branch of the median nerve e. Severance of musculocutaneous nerve

Odontoid process

A forensic pathologist is examining a set of skeletal remains unearthed from what was apparently a shallow grave. In reconstructing the skeleton, he identifies the individual vertebrae and aligns them in sequence. Which of the following is a distinguishing feature of the second cervical (C2) vertebra? A. Absence of the vertebral body B. Articular facets that face medio-lateral C. Costal facets on the vertebral body D. Odontoid process E. Trifid spinous process

It is a fracture of the distal end of the radius

A large dog knocked down a 79-year-old woman. Trying to ease her fall, she outstretched her upper limb. When she sat up, she noticed that her "wrist" (distal forearm) was deformed and very painful. She was taken to a hospital where the orthopedic surgeon said she had a Colles fracture. Which of the following statements about a Colles fracture is correct? a. It is a fracture of the distal end of the radius. b. It is fracture of the distal ulna. c. The fracture commonly results from forced flexion of the hand. d. The distal fragment of the fractured bone is displaced anteriorly. e. In this fracture, the distal end of the radius is located distal to that of the ulna.

Musculocutaneous nerve

A physician tests the myotatic biceps reflex as shown. A normal response of involuntary contraction of the biceps brachii muscle is noted. This reflex confirms the integrity of what nerve? A. Axillary nerve B. Median nerve C. Ulnar nerve D. Radial nerve E. Musculocutaneous nerve

Supraspinatus tendonitis

A right-handed 21-year-old college student visits his physician because of pain in his right shoulder that developed after starting a summer job on a construction crew 2 weeks ago. He explains that on his job site he regularly lifts heavy construction materials over his head. During physical examination, the patient experiences sharp pain in the range of 80 to 150 degrees of abduction at the glenohumeral joint. What is the most likely diagnosis? a. Infraspinatus tendonitis b. Supraspinatus tendonitis c. Acromioclavicular (AC) joint arthritis d. Degenerative arthritis of the shoulder e. Broken clavicle

Flattened thenar eminence

A secretary comes in to your office complaining of pain in her wrists from typing all day. You determine that she likely has carpal tunnel syndrome. Which of the following conditions would help you determine the diagnosis? a. Flattened thenar eminence b. Inability to adduct the little finger c. Inability to flex the DIP joint of the ring finger d. Loss of skin sensation of the medial one and one-half fingers e. Atrophied adductor pollicis muscle

A 32-year old patient has a tension pneumothorax that can be treated with needle aspiration. To avoid an injury of the intercostal neurovascular bundle, the needle may be inserted in which of the following locations?

Above the upper border of the ribs

Ulnar nerve

After winning a boxing match, a 24-year-old man is unable to abduct his fingers. Which of the following nerves is injured? a. Ulnar nerve b. Median nerve c. Radial nerve d. Musculocutaneous nerve e. Axillary nerve

Suprascapular and axillary

An 18-year-old boy involved in an automobile accident presents with arm that cannot abduct. His paralysis is caused by damage to which of the following nerves? a. Suprascapular and axillary b. Thoracodorsal and upper subscapular c. Axillary and musculocutaneous d. Radial and lower subscapular e. Suprascapular and dorsal scapular

Trapezius, scapula, latissimus dorsi

An internal medicine attending physician asks a medical student to place the bell of her stethoscope on the triangle of auscultation to hear a patient's breathing sounds. Which of the following structures make up the boundaries of this triangle? A. Deltoid, latissimus dorsi, scapula B. Trapezius, scapula, latissimus dorsi C. Latissimus dorsi, ilium, external abdominal oblique D. Rhomboids, levator scapulae, splenius capitis E. Longissimus, rhomboids, vertebral spinous processes

30. A 75-year-old veteran suffers a heart attack and is found in his home unconscious. He is in ventricular tachycardia and is shocked into normal sinus rhythm. He undergoes emergent catheterization and is found to have a thrombosis in the coronary sinus. Which of the following cardiac veins might remain normal in diameter by catheterization study by the cardiologist?

Anterior cardiac vein

22. A 17-year-old boy is involved in a group fight, and a stab wound lacerates a ventral root of his thoracic spinal nerve. Cell bodies of the injured nerve fibers are located in which of the following nervous structures?

Anterior horn and lateral horn of spinal cord

Injury to Axillary Nerve

Atrophy of the deltoid occurs when the axillary nerve (C5 and C6) is severely damaged (e.g., as might occur when the surgical neck of the humerus is fractured). As the deltoid atrophies unilaterally, the rounded contour of the shoulder disappears, resulting in visible asymmetry of the shoulder outlines. This gives the shoulder a flattened appearance and produces a slight hollow inferior to the acromion. A loss of sensation may occur over the lateral side of the proximal part of the arm, the area supplied by the superior lateral cutaneous nerve of the arm. To test the deltoid (or the function of the axillary nerve) the arm is abducted, against resistance, starting from approximately 15 degrees.

. 1. A young man is brought to the emergency department after being mugged. He was stabbed in the shoulder after refusing to give his wallet to his assailant. If the stab wound lacerated the posterior humeral circumflex artery passing through the quadrangular space on the shoulder region, which of the following nerves might be injured?

Axillary nerve

23. A race car driver is brought to the city trauma center after a high-speed crash in which his car spun out of control and struck the concrete embankment. He has blunt trauma to his chest and undergoes extensive vascular studies to determine which blood vessels are still intact. The interventional radiologist recalls that one of the following veins drains directly into the superior vena cava. Which vein would this be?

Azygos vein

3. A biomedical engineer would like to reconstruct the arm of a boy who underwent amputation to treat a life-threatening infection. In designing the prosthetic arm, the engineer will need to know that which of the following muscles flexes the elbow and is innervated by the radial nerve?

Brachioradialis

16. Deep venous thrombosis is a common complication from sitting in one position for a prolonged duration, such as during a long car trip or a long plane flight. The first vascular channels likely to be obstructed or occluded by an embolus from the deep veins of a lower limb are the:

Branches of the pulmonary arteries

What is the name of the keel-like septum at the bifurcation of the trachea?

Carina

B

Choose the appropriate lettered site or structure in the following radiograph of the wrist and hand (see figure). If the floor of the anatomic snuffbox and origin of the abductor pollicis brevis are damaged, which of the following bones is most likely to be involved? A B C D E

A

Choose the appropriate lettered structure in the radiograph of the bones of the hand (see figure). Fracture of which bone may cause a deep tenderness in the anatomic snuffbox? A B C D E

C

Choose the appropriate lettered structure in the radiograph of the bones of the hand (see figure). Fracture of which bone may cause paralysis of the flexor digiti minimi and opponens digiti minimi muscles? A B C D E

19. A 21-year-old man falls from the attic and is brought to the emergency department. Examination and radiogram reveal that the lateral longitudinal arch of his foot is flattened. Which of the following bones is displaced?

Cuboid

20. A 72-year-old woman with Parkinson's disease fell down in the bathtub at her home and suffered a dislocation of the hip joint that may result in vascular necrosis of the femoral head and neck because of injuries to the arteries. Which of the following arteries might remain intact?

Deep iliac circumflex artery

29. A 31-year-old NHL hockey player complains of numbness in the area of his umbilicus after the national championship game. Which of the following structures that carries GSA fibers was injured during the hockey game?

Dorsal root

15. A man interviewing for a new administrative position as hospital chief executive officer notices difficulty walking after sitting with his leg crossed for 2 hours. He was nervous during the interview but even more so now that he is attempting to stand to follow two board members for a tour of the hospital. Which of the following actions is most seriously affected by compression and temporary paralysis of the deep peroneal nerve?

Dorsiflexion of the foot

Between the atlas and the axis

During a general physical examination, a physician asks her patient to stand upright facing straight ahead, then to look to the side with the chin in line with the shoulder. Most of the lateral rotation responsible for this movement occurs at which of the following locations? A. At the atlanto-occipital joints B. At the lower lumbar joints C. Between the atlas and the axis D. Between the lower cervical vertebrae E. Between the upper thoracic vertebrae

Flexor Pollicis Brevis

Flexor Reinaculum and tibercles of scaphoid and trapezium, lateral side of base of proximal phalanx of thumb, Flexes thumb, Recurrent branch of median nerve (C8, T1), Superficial palmar branch of the radial artery

Brachialis

Flexes forearm in all positions, Musculocutaneous Nerve (C5, C6), Brachial Artery

Opponens Pollicis

Flexor Reinaculum and tibercles of scaphoid and trapezium, Lateral side of 1st metacarpal, oppose thumb, it draws 1st metacarpal medially to center palm and rotate medially, Recurrent branch of median nerve (C8, T1), Superficial palmar branch of the radial artery

The superior parts of his brachial plexus

Following a motorcycle accident,your 22-year-old patient who you are seeing in the ED is complaining of neck/shoulder pain. The accident involved a forced abnormal increase in the angle between his neck and shoulder. His upper limb movements are clearly not normal and when he lets his upper limb hang, it is adducted at his side and in medial rotation. You suspect injury to which of the following? a. The superior parts of his brachial plexus b. The inferior parts of his brachial plexus c. The posterior cord of his brachial plexus d. The axillary nerve e. His scalene muscles

Brachial Plexus Block

Injection of an anesthetic solution into or immediately surrounding the axillary sheath interrupts nerve impulses and produces anesthesia of the structures supplied by the branches of the cords of the plexus. Combined with an occlusive tourniquet technique to retain the anesthetic agent, this procedure enables surgeons to operate on the upper limb without using a general anesthetic. The brachial plexus can be anesthetized using a number of approaches, such as interscalene, supraclavicular, and axillary.

Which of the following is correct based on the above image, which was taken during inspiration?

It shows a patient who likely has paralyzed right phrenic nerve

7. A 42-year-old woman presents to an out-patient clinic with a 6-month history of numbness and tingling on the palmar aspect of her lateral three and one-half fingers, loss of pronation, and flattening of the thenar eminence. Injury to which of the following nerves could cause such a condition?

Median nerve

13. The secretary of a rather verbose academic physician in internal medicine complains of numbness and tingling in her hands and fingers. She is constantly typing long patient visit dictations and now has carpal tunnel syndrome, which is due to compression of which of the following structures?

Mediannerve

A 7 day old baby is diagnoses as having congenital neonatal emphysema, which is caused by collapsed bronchi because of failure of bronchial cartilage development. Bronchial cartilages are derived from which of the following derivations.

Mesoderm

18. During a sports medicine physical by a local family physician, a young woman is tested for stability of her joints before tryouts for the high school team. Which of the following ligaments is important in preventing forward displacement of the femur on the tibia when the weight-bearing knee is flexed?

Posterior cruciate ligament

Which of the following is a branch of the right coronary artery?

Posterior interventricular artery

28. A stab wound penetrates the posterior thoracic wall near vertebra of a 24-year-old man. Examination at the emergency department indicates a lesion of gray rami communicantes. Which of the following nerve fibers would most likely be damaged?

Postganglionic parasympathetic fibers

14. While playing in a Super Bowl game, a 32-year-old professional football player is tackled, and his anterior cruciate ligament is torn. If not injured, the anterior cruciate ligament of the knee joint:

Resists posterior displacement of the femur on the tibia

26. A 21-year-old woman comes to the emergency department with acute chest pain and shortness of breath. Her chest radiograph shows opacification of one of her lungs. She undergoes thoracentesis, which reveals that she has a chylothorax resulting from rupture of the thoracic duct. Lymphatic drainage remains normal in which of the following areas?

Right thorax

A 54-year old high school teacher has been diagnosed with coronary artery disease and is going to undergo coronary bypass surgery. During the surgery, the thoracic surgeon decides to use the internal thoracic artery for one of the bypasses. Which of the following arteries gives rise to the internal thoracic artery

Subclavian artery

A chest X-ray reveals lobar pneumonia located over the horizontal fissure of the lung. Which of the following lobes of the lung would inflamed and full of fluid?

Superior lobe of right lung

9. A 23-year-old man falls from a ladder and injures his arm. On examination, he feels numbness and has no sweating on the lateral side of his forearm, indicating damage to the lateral antebrachial cutaneous nerve. The cell bodies of injured nerve fibers involved in sweating are located in which of the following structures?

Sympathetic chain ganglia

A 40-year-old man goes to his family physician for an annual examination. Which of the following locations is ideal for placement of the stethoscope for auscultation of the tricuspid valve of the heart

The ideal placement of the stethoscope for auscultation of the tricuspid valve is at the 5th intercostal space slightly to the left of the sternal border. The tricuspid valve separates the right atrium and the right ventricle.

Deltoid

The physical therapist is testing the action of one of your patient's muscles by bringing the patient's upper limb away from her side and then asking the patient to abduct against resistance. What is the muscle being tested? A. Deltoid B. Pectoralis major C. Pectoralis minor D. Latissimus dorsi E. Serratus anterior

Which of the following is correct for the bronchial arteries?

They supply lung tissue including bronchi

27. An elderly man is choking on his food at a restaurant and attempts by other patrons to dislodge the food bolus using the Heimlich procedure have failed. A retired anesthesiologist rushes to his table and prepares for emergent tracheostomy. She locates the manubrium of the sternum and recalls that it is free from articulation with which of the following structures?

Third rib

Which of the following congenital defects is the direct outcome of the malformation of the spiral partitioning of the conus cordis and truncus arteriosus?

Transposition of the great vessels

10. There are only 30 minutes left before the concert starts. The pianist, who incidentally has a cut over the palmar surface of her wrist, notices that she is unable to pick up a piece of music between her index and middle fingers. Which of the following nerves is most likely damaged?

Ulnar

11. A 29-year-old carpenter receives a crush injury to his metacarpophalangeal joint of the fourth digit (ring finger) while remodeling his neighbor's porch. Which of the following pairs of nerves innervates the muscle that moves the injured joint?

Ulnar and radial nerves

6. After winning a boxing match, a 24-year-old man is unable to abduct his fingers. Which of the following nerves is injured?

Ulnar nerve

Has not begun to walk upright

You are examining an abandoned infant that you estimate to be close to 1 year of age. In evaluating the infant, you notice that there is no lumbar lordosis. What may this tell you about the infant? A. Has spondylolysis B. Has spina bifida occult C. Has not begun to walk upright D. Has hypertrophy of the erector spinae E. Has spasticity in his transversospinalis muscles

Scaphoid

You are examining the hand of a patient after a fall in which she landed on her outstretched hand. When you palpate deeply in the floor of the anatomical snuff box, you are most likely feeling which of the following? A. Radial styloid process B. Capitate C. Scaphoid D. Lunate E. Pisiform

C7

You are running your finger down the spinous processes of your 34-year-old patient and find the most prominent one. This is known as the vertebra prominens and is typically the spinous process of which of the following? A. C2 B. C5 C. C7 D. T2 E. L1

Radial

You are seeing a patient in the ED after a skiing accident in which she sustained a mid-humeral shaft fracture. She is displaying a wrist-drop. You suspect injury to which of the following nerves? A. Median B. Ulnar C. Radial D. Musculocutaneous E. Axillary

Dislocated shoulder

You are treating a patient in the ED who is supporting his right upper limb with his left hand. A radiograph of his shoulder is shown above. What is your diagnosis? A. Separated shoulder B. Dislocated shoulder C. Fractured coracoid process D. Fractured clavicle E. Normal shoulder

Serratus anterior

You are working in the ED treating a 34-year-old patient who was in a knife fight and has a wound under his right axilla. He seems unable to fully abduct his arm above horizontal. When you ask him to press against the wall you notice that he has a "winged scapula". Which muscle is likely paralyzed? A. Pectoralis major B. Rhomboid major C. Trapezius D. Serratus anterior E. Latissimus dorsi

Test for sensation on the dorsal aspect of the proximal lateral hand

You suspect the patient you are examining in the ED has a minor radial nerve injury that primarily has affected the superficial branch of this nerve. To test the function of this superficial branch, you would do which of the following? A. Test for sensation on the dorsal aspect of the proximal lateral hand B. Test for sensation on the ventral aspect of the proximal lateral hand C. Test for sensation on the ventral aspect of the central palm D. Test the power of thumb extension E. Test the power of thumb adduction

Ulnar

Your 12-year-old patient fell on his elbow and now has severe pain and deformity at the medial epicondyle of the humerus. A CT image shown above shows a displaced fracture (arrows). Based on this radiograph, you would most likely be concerned about damage to which of the following nerves? A. Radial B. Ulnar C. Median D. Axillary E. Thoracodorsal

Ulnar

Your 12-year-old patient fell on his elbow and now has severe pain and deformity at the medial epicondyle of the humerus. A CT image shown above shows a displaced fracture (arrows). Based on this radiograph, you would most likely be concerned about damage to which of the following nerves? a. Radial b. Ulnar c. Median d. Axillary e. Thoracodorsal

Fifth metacarpal fracture

Your 17-year-old male patient was in a fist fight in a bar and now has pain along the medial side of his distal hand, along with flexion deformity (caused by volar angulation of the distal fracture fragment). A radiograph of his hand is shown above. What is your diagnosis? a. Colles fracture b. Fracture of the triangular ligament c. Fifth metacarpal fracture d. Fracture of first phalange of fifth digit e. Dislocation of hamate-fifth metacarpal joint

The extensor digitorum tendon to the distal phalanx of the index finger was avulsed

Your 23-year-old baseball-playing patient misjudged a ball he was trying to catch. The ball acutely hyperflexed the distal interphalangeal joint of his index finger, which he is now unable to extend voluntarily. You suspect which of the following? a. The flexor digitorum superficialis tendon to the distal phalanx of the index finger was avulsed b. The flexor digitorum profundus tendon to the distal phalanx of the index finger was avulsed c. The extensor digitorum tendon to the distal phalanx of the index finger was avulsed d. The distal interphalangeal joint of that finger dislocated e. The collateral ligaments of the distal interphalangeal joint of that finger were sprained

Teres major

Your 45-year-old patient has frequent humeral dislocations. You refer him to a physical therapist for exercises to strengthen his rotator cuff muscles. Which of the following is not part of this group of muscles? A. Supraspinatus B. Teres major C. Teres minor D. Infraspinatus E. Subscapularis

Brachioradialis

Your 56-year-old patient is showing weak wrist flexion. Weakness in which of the following muscles would not be expected to be associated with any wrist flexion weakness? A. Brachioradialis B. Flexor carpi radialis C. Flexor carpi ulnaris D. Flexor digitorum superficialis E. Flexor digitorum profundus

Flexor Carpi Radialis

medial epicondyle of humerus, base of the 2nd (3rd) metacarpal bone, Flexes and abducts hand (at wrist), median nerve C6, C7, Ulnar artery

The expansion of the lactiferous duct, near the nipple, is the

lactiferous

24. A 58-year-old stockbroker is brought to the cardiac catheterization laboratory emergently after evaluation in the emergency department has determined that he is suffering from an acute myocardial infarction. During the catheterization, he is found to have inadequate blood flow in the artery that runs aside the great cardiac vein in the anterior interventricular sulcus of the heart. This is most likely an acute occlusion of the:

left coronary artery

The attending faculty of the coronary intensive care unit demonstrates to his students a normal heart examination. The first heart sound is produced by near-simultaneous closure of which of the following valves

tricuspid and mitral

There are only 30 minutes left before the concert starts. The pianist, who incidentally has a cut over the palmar surface of her wrist, notices that she is unable to pick up a piece of music between her index and middle fingers. Which of the following nerves is most likely damaged?

ulnar nerve. This pianist is unable to pick up a music piece between the index and middle fingers because she cannot adduct her index finger and abduct the middle finger. The adductor of the index finger is the palmar interosseous muscle, which is innervated by the ulnar nerve. Abductors of the middle finger are dorsal interosseous muscles, which are innervated by the ulnar nerve. The other nerves do not innervate adductors or abductors of the fingers.

Extensor Pollicis Longus and Brevis

"Longus: Posterior surface of middle third of ulna and interosseous membrane Brevis: Posterior surface of distal third of radius and interosseous membrane", Longus: Dorsal aspect of base of distal phalanx of thumb / Brevis: Dorsal aspect of base of proximal phalanx of thumb, "Longus: Extends distal phalanx of thumb at metacarpophalangeal joint; extends carpometacarpal joint Brevis: Extends proximal phalanx of thumb at metacarpophalangeal joint; extends carpometacarpal joint", posterior interossesous nerve (C7, C8), continuation of deep branch of radial nerve, Posterior interosseous artery

Extensor Digitorum

Lateral epicondyle of humerus (common extensor origin), Extensor explansions of medial four fingers, Extends medial four fingers primarily at metacarpophalangeal joints, secondariliy at interphalangeal joints, Posterior Interosseous nerve (C7, C8), continuation of deep branch of radial nerve, Posterior Interosseous Artery

Lateral two lumbricals and opponens pollicis

A 27-year-old pianist with a known carpal tunnel syndrome experiences difficulty in finger movements. Which of the following intrinsic muscles of her hand is paralyzed? a. Lateral two lumbricals and opponens pollicis b. Palmar interossei and adductor pollicis c. Dorsal interossei and lateral two lumbricals d. Abductor pollicis brevis and palmar interossei e. Medial two and lateral two lumbricals

Flexor digitorum superficialis

A patient comes in complaining that she cannot flex her proximal interphalangeal joints. Which of the following muscles appear(s) to be paralyzed on further examination of her finger? a. Flexor digitorum superficialis b. Palmar interossei c. Dorsal interossei d. Flexor digitorum profundus e. Lumbricals

Superior thoracic

A patient comes in with a gunshot wound and requires surgery in which his thoracoacromial trunk needs to be ligated. Which of the following arterial branches would maintain normal blood flow? a. Superior thoracic b. Acromial c. Pectoral d. Clavicular e. Deltoid

Radial and musculocutaneous nerves

A 22-year-old female Macarena dancer fell from the stage and complains of elbow pain and inability to supinate her forearm. Which of the following nerves are most likely injured from this accident? a. Radial and musculocutaneous nerves b. Median and ulnar nerves c. Axillary and radial nerves d. Ulnar and axillary nerves e. Musculocutaneous and median nerves

Extensor Carpi Ulnaris

Lateral epicondyle of humerus; posterior border of ulna via a shared aponerousis, Dorsal aspect of base of 5th metacarpal, Extends and adducts hand at wrist joint (also active during fist clenching), Posterior Interosseous nerve (C7, C8), continuation of deep branch of radial nerve, Posterior interosseous artery

Dorsal root ganglion

A 23-year-old man complains of numbness on the medial side of the arm following a stab wound in the axilla. On examination, he is diagnosed with an injury of his medial brachial cutaneous nerve. In which of the following structures are the cell bodies of the damaged nerve involved in numbness located? a. Dorsal root ganglion b. Sympathetic chain ganglion c. Anterior horn of the spinal cord d. Lateral horn of the spinal cord e. Posterior horn of the spinal cord

Dorsal root ganglia

A 10-year-old boy falls off his bike, has difficulty in moving his shoulder, and is brought to an emergency department. His radiogram and angiogram reveal fracture of the surgical neck of his humerus and bleeding from the point of the fracture. The damaged nerve causes numbness of the lateral side of the arm. Cell bodies of the injured nerve fibers involved in sensory loss are located in which of the following structures? a. Dorsal root ganglia b. Anterior horn of the spinal cord c. Posterior horn of the spinal cord d. Lateral horn of the spinal cord e. Sympathetic chain ganglia

Infraspinatus

A 16-year-old boy attempts to lift a heavy barbell in order to impress his high school football coach. During the lift, violent contraction of the shoulder muscles causes avulsion of the greater tuberosity of the left humerus. Which of the following muscles is attached to the displaced bone fragment? a. Deltoid b. Infraspinatus c. Pectoralis major d. Subscapularis e. Teres major

Torn coracoclavicular ligament

A 17-year-old boy comes to the emergency room after a hard fall onto the lateral aspect of his left shoulder during a high school basketball game. He complains of generalized pain during shoulder motion. On physical examination, the distal end of the clavicle is prominent and distinctly palpable. Radiological findings confirm the diagnosis of a severe (grade 3) shoulder separation. Which of the following features is a component of this condition? a. Dislocated head of the humerus b. Torn coracoclavicular ligament c. Fractured clavicle d. Dislocated sternal end of the clavicle e. Torn anterior glenohumeral (GH) ligament

Radial nerve

A 17-year-old boy is injured in an automobile accident. He has a fracture of the shaft of the humerus. Which of the following nerves is most likely damaged? a. Radial nerve b. Axillary nerve c. Musculocutaneous nerve d. Median nerve e. Ulnar nerve

Posterior circumflex humeral artery

A 17-year-old boy is taken to the emergency room after being struck on the lateral aspect of the left shoulder by a baseball bat during a gang fight. Examination reveals a fracture of the surgical neck of the left humerus, displaced bone fragments piercing the quadrangular space of the shoulder, and deep blood accumulation around the fracture. Which of the following blood vessels was most likely severed? a. Cephalic vein b. Circumflex scapular artery c. Posterior circumflex humeral artery d. Profunda brachii artery e. Subscapular artery

Fracture of the fifth metacarpal bone

A 17-year-old man has pain and moderate swelling over the dorsomedial aspect and in the hypothenar area of his right hand after punching a locker over a dispute with his girlfriend. What is the most likely finding on an X-ray of his hand? a. Dislocation of the fifth metacarpophalangeal joint b. Fracture of the triquetral bone c. Fracture of the proximal phalanx of the ring finger d. Fracture of the proximal phalanx of the little finger e. Fracture of the fifth metacarpal bone

Ulnar nerve

A 31-year-old professional baseball pitcher suffers a torn medial collateral ligament of the elbow in his throwing arm. Two weeks later, he undergoes "Tommy John surgery," in which the tendon of the palmaris longus muscle is transplanted into the elbow to reconstruct the damaged ligament. During the operative procedure, the surgeon must be keenly aware of the close relationship of which of the following structures to the medial collateral ligament? a. Brachial artery b. Cephalic vein c. Radial nerve d. Ulnar artery e. Ulnar nerve

Scaphoid

A 37-year-old factory worker fractures multiple bones distal to the elbow when his hand and forearm are crushed by equipment dropped by a faulty hydraulic lift. Which of the following bones, if fractured, would most likely develop avascular necrosis? a. Distal radius b. Midshaft ulna c. Fifth metacarpal d. Lunate e. Scaphoid

Weakness extending the wrist and fingers

A 48-year-old woman falls on an icy sidewalk and lands on her right elbow. She suffers a midshaft humeral fracture, as seen on the given X-ray. The attending physician wants to assess whether the nerve residing in the spiral groove of the humerus is damaged. What sign or symptom would confirm damage to this nerve? a. Numbness on the lateral (radial) aspect of the forearm b. Numbness of the medial aspect of the upper arm c. Numbness over the superolateral aspect of the upper arm d. Weakness extending the wrist and fingers e. Weakness in grip strength

Anterior axillary lymph nodes

A 48-year-old woman is diagnosed with a malignant tumor in the superolateral quadrant of the right breast, including the axillary tail. If it metastasizes, this cancer will most likely spread first to which of the following locations? a. Lateral axillary lymph nodes b. Anterior axillary lymph nodes c. Deep cervical lymph nodes d. Parasternal lymph nodes e. Contralateral breast lymph nodes

Upper trunk of the brachial plexus

A 50-year-old man falls off a ladder while cleaning his windows, landing on the ground as seen in the given drawing. He does not seek medical aid, believing his general soreness will go away with time. However, after several months, he develops a postural deformity of his left upper limb that includes an adducted, medially rotated, and extended shoulder, extended elbow, and pronated forearm. The injury and subsequent condition reflect damage to what structure? a. Medial cord of the brachial plexus b. Lateral root of the median nerve c. Lower roots of the brachial plexus d. Posterior divisions of the brachial plexus e. Upper trunk of the brachial plexus

Ulnar nerve in the wrist

A 52-year-old retired professional cyclist, who still rides his bike 400 miles per week, comes to his physician complaining of hand problems. The physician notes hyperextension of the ring and little fingers at the metacarpophalangeal joints and flexion at the interphalangeal joints within the same fingers (see photo). During examination, the patient has no weakness in flexion or adduction of the wrist. What nerve is compressed at what location? a. Ulnar nerve in the elbow b. Ulnar nerve in the wrist c. Median nerve in the wrist d. Median nerve in the elbow e. Median nerve in the axilla

Ulnar nerve in the elbow

A 56-year-old woman was stopped at a light when her car was rear-ended by another car. She had her right arm on the steering wheel, and the impact caused forced flexion at her elbow. Several months later, she comes to her physician complaining of numbness and a "pins and needles" sensation in her right little finger when she talks on the phone, rests her head on her right hand at work, or spends most of her day typing at work. She also notices the quality of her typing and her ability to play the violin have diminished. Which nerve is compressed at what location? a. Ulnar nerve in the elbow b. Ulnar nerve in the wrist c. Median nerve in the wrist d. Median nerve in the elbow e. Median nerve in the axilla

Abduction of the index finger

A 62-year-old man is diagnosed with a Pancoast's tumor that invades the inferior trunk of the brachial plexus. Which of the following muscle actions most likely resulted from injury to the brachial plexus? a. Abduction of the index finger b. Lateral rotation of the arm c. Extension of the ring finger d. Flexion of the forearm e. Pronation of the forearm

Latissimus dorsi

A 65-year-old man is brought to the emergency room after being attacked in his office by a disgruntled co-worker. The attacker reportedly used a long, narrow-bladed letter-opener to inflict multiple stab wounds to the man's back. Physical examination shows a puncture wound in the posterior axillary fold. The patient presents with weakness in extension, adduction, and medial rotation of his arm. Which of the following muscles is most likely cut in this injury? a. Pectoralis minor b. Latissimus dorsi c. Levator scapulae d. Serratus anterior e. Teres minor

Extension of the arm

A ballet dancer falls to the floor and hurts herself during a practice session before opening night. She sustains an injury to the thoracodorsal nerve that would probably affect the strength of which of the following movements? a. Extension of the arm b. Adduction of the scapula c. Elevation of the scapula d. Abduction of the arm e. Lateral rotation of the arm

Ulnar artery and ulnar nerve

A man injures his wrist on broken glass. Which of the following structures entering the palm superficial to the flexor retinaculum may be damaged? a. Ulnar artery and ulnar nerve b. Ulnar nerve and median nerve c. Median nerve and flexor digitorum profundus d. Median nerve and flexor pollicis longus e. Ulnar nerve and flexor digitorum superficialis

Radial Nerve Injury

Although the radial nerve supplies no muscles in the hand, radial nerve injury in the arm by a fracture of the humeral shaft can produce serious disability of the hand. This injury is proximal to the branches to the extensors of the wrist, so wrist drop is the primary clinical manifestation. The hand is flexed at the wrist and lies flaccid, and the digits also remain in the flexed position at the metacarpophalangeal joints. The extent of anesthesia is minimal, even in serious radial nerve injuries, and usually is confined to a small area on the lateral part of the dorsum of the hand. Severance of the deep branch results in an inability to extend the thumb and the metacarpophalangeal joints of the other digits. Loss of sensation does not occur because the deep branch is entirely muscular and articular in distribution.

Flexor Pollicis Longus

Anterior surface of radius and adjacent interosseous membrane, Base of stal phalanx of thumb, Flexes phalanges of 1st digit (thumb), Anterior interosseous nerve, from median nerve (C8, T1), Radial artery and Anterior interosseous artery

Carpal Tunnel Syndrome

Carpal tunnel syndrome results from any lesion that significantly reduces the size of the carpal tunnel or, more commonly, increases the size of some of the structures (or their coverings) that pass through it (e.g., inflammation of the synovial sheaths). The median nerve is the most sensitive structure in the carpal tunnel and, therefore, is the most affected. The median nerve has two terminal sensory branches that supply the skin of the hand; hence paresthesia (tingling), hypothesia (diminished sensation), or anesthesia (absence of tactile sensation) may occur in the lateral three and a half digits. Recall, however, that the palmar cutaneous branch of the median nerve arises proximal to and does not pass through the carpal tunnel; thus sensation in the central palm remains unaffected. This nerve also has one terminal motor branch, the recurrent branch, which innervates the three thenar muscles.

Tricep Brachii

Chief extensor of forearm; Long Head extends arm and resists dislocation of humerus (Especially during abdcution), Radial Nerve (C6, C7, C8)

B

Choose the appropriate lettered structure in the radiograph of the bones of the hand (see figure). Which bone articulates with the radius and triquetrum?

Compression of Axillary Artery

Compression of the third part of the axillary artery against the humerus may be necessary when profuse bleeding occurs. If compression is required at a more proximal site, the axillary artery can be compressed at its origin at the lateral border of the first rib by exerting downward pressure in the angle between the clavicle and the attachment of the SCM.

Dupuytren Contracture of Palmar Fascia

Dupuytren contracture is a disease of the palmar fascia resulting in progressive shortening, thickening, and fibrosis of the palmar fascia and palmar aponeurosis. The fibrous degeneration of the longitudinal digital bands of the aponeurosis on the medial side of the hand pulls the fourth and fifth fingers into partial flexion at the metacarpophalangeal and proximal interphalangeal joints. The contracture is frequently bilateral. Treatment of Dupuytren contracture usually involves surgical excision of all fibrotic parts of the palmar fascia to free the fingers.

The knife pierced the pectoralis minor muscle to reach the cords of the brachial plexus.

During an attempted robbery, a 20-year-old man of average body build is stabbed once with a long, narrow knife. Initial examination in the emergency room indicates the knife penetrated the anterior wall of the right axilla and reached approximately the center of the axillary space. Which of the following is most likely true of this injury? a. The knife penetrated the teres major muscle. b. The knife pierced the pectoralis minor muscle to reach the cords of the brachial plexus. c. The knife probably struck one of the trunks of the brachial plexus. d. The suprascapular nerve is probably lesioned. e. The man will likely develop a "winged scapula" condition.

It is normal for contrast material to enter the subscapular bursa

During arthrography (injection of intra-articular radiographic contrast material) of the shoulder, which of the following is correct? a. It is normal for contrast material to enter the subacromial bursa b. It is normal for contrast material to enter the subscapular bursa c. It is normal for contrast material to enter the subscapularis tendon d. It is normal for contrast to enter the acromioclavicular joint e. It is normal for contrast to enter the scapulothoracic joint

Abductor Pollicis Brevis

Flexor Reinaculum and tibercles of scaphoid and trapezium, lateral side of base of proximal phalanx of thumb, abducts thumb, helps oppose, Recurrent branch of median nerve (C8, T1), Superficial palmar branch of the radial artery

Fracture of the hamate

Fracture of the hamate may result in nonunion of the fractured bony parts because of the traction produced by the attached muscles. Because the ulnar nerve is close to the hook of the hamate, the nerve may be injured by this fracture, causing decreased grip strength of the hand. The ulnar artery may also be damaged when the hamate is fractured.

Fracture of the Scaphoid

Fracture of the scaphoid often results from a fall on the palm with the hand abducted. The fracture occurs across the narrow part ("waist") of the scaphoid. Pain occurs primarily on the lateral side of the wrist, especially during dorsiflexion and abduction of the hand. Initial radiographs of the wrist may not reveal a fracture, but radiographs taken 10 to 14 days later reveal a fracture because bone resorption has occurred. Owing to the poor blood supply to the proximal part of the scaphoid, union of the fractured parts may take several months. Avascular necrosis of the proximal fragment of the scaphoid (pathological death of bone resulting from poor blood supply) may occur and produce degenerative joint disease of the wrist.

Fractures of Ulna and Radius

Fractures of both the ulna and the radius are the result of severe injury. A direct injury usually produces transverse fractures at the same level, often in the middle third of the bones. Because the shafts of these bones are firmly bound together by the interosseous membrane, a fracture of one bone is likely to be associated with dislocation of the nearest joint. Fracture of the distal end or the radius is the most common fracture in people older than 50 years. A complete fracture of the distal 2 cm of the radius, called a Colles fracture, is the most common fracture of the forearm. The distal fragment of the radius is displaced dorsally and often comminuted (broken into pieces). The fracture results from forced dorsiflexion of the hand, usually as the result of trying to ease a fall by outstretching the upper limb. Often, the ulnar styloid process is avulsed (broken off). Normally, the radial styloid process projects farther distally than the ulnar styloid process; consequently, when a Colles fracture occurs, this relationship is reversed because of shortening of the radius. This clinical condition is often referred to as a dinner fork (silver fork) deformity because a posterior angulation occurs in the forearm just proximal to the wrist and the normal anterior curvature of the relaxed hand. The posterior bending is produced by the posterior displacement and tilt of the distal fragment of the radius.

Coracobrachialis

Helps flex and adduct arm; resists dislocation of shoulder, Musculocutaneous Nerve (C5, C6. C7), Brachial Artery

Flexor Digiti Minimi

Hook of Hamate and flexor retinaculum, Medial side of base of proximal phalanx of 5th finger, Flexes proximal phalanx of 5th finger, Deep branch of Ulnar Nerve (C8, T1), ulnar artery

Flexor digitorum superficialis

Humero-ulnar Head: medial epicondyle of humerus and coronoid process of ulna / Radial Head: Oblique line of radius, Shafts (bodies) of middle phalanges of medial four digits, Flexes proximal interphalangeal joints of middle four digits, acting more strongly, it also flexes proximal phalanges at metacarpophalangeal joints, median nerve C7, C8, T1, Radial and Ulnar artery

Extensor pollicis longus

In examining a 24-year-old patient after a hand injury, you want to ensure the integrity of the muscles that border the anatomical snuff box. You ask your patient to abduct his thumb and extend his other fingers and you note that the tendon that forms the medial border of the snuff box is not evident and likely injured. This tendon belongs to which of the following muscles? a. Extensor pollicis longus b. Extensor pollicis brevis c. Abductor pollicis longus d. Abductor pollicis brevis e. Brachiordialis

Rotator Cuff Injuries and Supraspinatus

Injury or disease may damage the rotator cuff, producing instability of the glenohumeral joint. Rupture or tear of the supraspinatus tendon is the most common injury of the rotator cuff. Degenerative tendinitis of the rotator cuff is common, especially in older people. These syndromes are discussed in detail later in this chapter, in relationship to the glenohumeral (shoulder) joint.

Injury to Musculocutaneous Nerve

Injury to the musculocutaneous nerve in the axilla is usually inflicted by a weapon such as a knife. A musculocutaneous nerve injury results in paralysis of the coracobrachialis, biceps, and brachialis; consequently, flexion of the elbow and supination of the forearm are greatly weakened. Loss of sensation may occur on the lateral surface of the forearm supplied by the lateral cutaneous nerve of the forearm.

Injury to Radial Nerve

Injury to the radial nerve superior to the origin of its branches to the triceps brachii results in paralysis of the triceps, brachioradialis, supinator, and extensor muscles of the wrist and fingers. Loss of sensation occurs in areas of skin supplied by this nerve. When the radial nerve is injured in the radial groove, the triceps is usually not completely paralyzed but only weakened because only the medial head is affected; however, the muscles in the posterior compartment of the forearm that are supplied by more distal branches of the radial nerve are paralyzed. The characteristic clinical sign of radial nerve injury is wrist-drop (inability to extend the wrist and fingers at the metacarpophalangeal joints). Instead, the wrist is flexed because of unopposed tonus of the flexor muscles and gravity.

Trauma to Median Nerve

Lesions of the median nerve usually occur in two places: the forearm and wrist. The most common site is where the nerve passes through the carpal tunnel. Laceration of the wrist often causes median nerve injury because this nerve is relatively close to the surface. This results in paralysis and wasting of the thenar muscles and the first two lumbricals. Hence opposition of the thumb is not possible and fine control movements of the second and third digits are impaired. Sensation is also lost over the thumb and adjacent two and a half fingers.

Palmaris Longus

Medial epicondyle of humerus, Distal half of flexor retinaculum; palmar aponeurosis, flexes hand (at wrist) and tenses palmar aponeurosis, median nerve C6, C7, Ulnar artery

Adductor Pollicis

Oblique Head: Bases of 2nd and 3rd metacarpals, capitate, adjacent carpals / Transverse Head: anterior surface of shaft of 3rd metacarpal, Medial side of base of proximal phalanx of thumb, adducts the thumb, Deep branch of Ulnar Nerve (C8, T1), Deep palmar arterial arch

flexor Carpi Ulnaris

Olecranon and posterior border (via aponeurosis), Pisiform, hook of hamate, 5th metacarpal, Flexes and adducts hand (at wrist), ulnar nerve C7, C8, Unlar Artery

Extensor Indicis

Posterior surface of distal third of ulna and interosseous membrane, extensor expansion of 2nd finger, Extends 2nd finger (enabling its independent extension); helps extend hand at wrist, posterior interossesous nerve (C7, C8), continuation of deep branch of radial nerve, Posterior interosseous artery

Abductor Pollicis Longus

Posterior surfaces of proximal halves of ulna, radius, and interosseous membrane, Base of 1st metacarpal, Adcuts thumb and extends it at carpometacarpal joint, posterior interossesous nerve (C7, C8), continuation of deep branch of radial nerve, Posterior interosseous artery

Brachioradialis

Proximal 2/3 of lateral supra-epicondylar ridge of humerus, Lateral surface of distal end of radius proximal to styloid process, Relatively weak flexion of forearm, maximal when forearm is in midpronated position, Radial Nerve (C5, C6, C7), Radial recurrent artery

Bicipital Aponeurosis

RUNS FROM BICEPS TENDON ACROSS THE CUBITAL FOSSA AND MERGES WITH ANTEBRACHIAL (DEEP) FASCIA COVERING FLEXOR MUSCLES IN THE MEDIAL SIDE OF FOREARM

carpals

Some Lovers Try Positions That They Cannot Handle S: scaphoid L: lunate T: triquetrum P: pisiform T: trapezium T: trapezoid C: capitate H: hamate

Synovial Cyst

Sometimes a nontender cystic swelling appears on the hand, most commonly on the dorsum of the wrist. The thinwalled cyst contains clear mucinous fluid. Clinically, this type of swelling is called a "ganglion" (G. swelling or knot). These synovial cysts are close to and often communicate with the synovial sheaths. The distal attachment of the extensor carpi radialis brevis (ECRB) tendon is a common site for such a cyst. A cystic swelling of the common flexor synovial sheath on the anterior aspect of the wrist can enlarge enough to produce compression of the median nerve by narrowing the carpal tunnel (carpal tunnel syndrome).

Biceps Brachii Long Head

Supinates forearm and when supinated: flexes forearm; flexes arm, Musculocutaneous Nerve (C5-C6), Brachial Artery

Biceps Brachii Short Head

Supinates forearm and when supinates: flexes forearm; flexes farm; Short head resists dislocation of shoulder, Musculocutaneous Nerve (C5-C6), Brachial Artery

Second part of the axillary artery

The lateral thoracic artery provides the main blood supply to the lateral side of the chest wall, including much of the breast. To deter excessive blood loss during a surgical procedure involving the breast, a surgeon can clamp the lateral thoracic artery near its origin. Which of the following arteries gives rise to this artery? a. First part of the axillary artery b. Second part of the axillary artery c. Third part of the axillary artery d. Third part of the subclavian artery e. First part of the brachial artery

Lateral to the tendon of the flexor carpi radialis

The pulse of the radial artery is readily palpable where the vessel passes which of the following structures? a. Across the anterior aspect of the lateral epicondyle of the humerus b. Between the tendons of the palmaris longus and flexor carpi ulnaris c. Lateral to the tendon of the flexor carpi radialis d. Superficial to the tendons of the extensor pollicis brevis and abductor pollicis longus e. Superficial to the carpal tunnel

Flexor digitorum profundus

Trigger finger is associated with swelling of the tendon(s) of which of the following muscles? a. Lumbricals b. Palmar interossei c. Flexor digitorum profundus d. Extensor digitorum e. Flexor carpi ulnaris

Pronator Teres

Ulnar head: Coronoid process of ulna; Humeral Head: Medial epicondyle of humerus, Middle of convexity of lateral surface of radius, Pronates and flexes forearm (at elbow), median nerve C6, C7, Radial and Ulnar artery

Ulnar Nerve Injury

Ulnar nerve injury usually occurs in one of four places: (1) posterior to the medial epicondyle of the humerus (most common), (2) in the cubital tunnel formed by the tendinous arch connecting the humeral and ulna r heads of the FCU, (3) at the wrist, and (4) in the hand. Ulnar nerve injury occurring at the elbow or wrist or in the hand may result in extensive motor and sensory loss to the hand. An injury to the nerve in the distal part of the forearm denervates most intrinsic hand muscles. The power of wrist adduction is impaired, and when an attempt is made to flex the wrist joint, the hand is drawn to the lateral side by the FCR in the absence of the "balance" provided by the FCU. After ulnar nerve injury, the person has difficulty making a fist because, in the absence of opposition, the metacarpophalangeal joints become hyperextended, and he or she cannot flex the fourth and fifth fingers at the distal interphalangeal joints when trying to make a fist.

Variations of Brachial Plexus

Variations in the brachial plexus formation are common. In addition to the five anterior rami (C5-T1) that form the roots of the plexus, small contributions may be made by the anterior rami of C4 or T2. When the superiormost root (anterior ramus) of the plexus is C4 and the inferiormost root is C8, it is called a prefixed brachial plexus. Alternatively, when the superior root is C6 and the inferior root is T2, it is a postfixed brachial plexus. In the latter type, the inferior trunk of the plexus may be compressed by the first rib, producing neurovascular symptoms in the upper limb. Variations also may occur in the formation of trunks, divisions, and cords; in the origin and/or combination of branches; and in the relationship to the axillary artery and scalene muscles.

Dislocation of the tendon of the long head of the biceps brachii

Yergason's test is a test for which a positive sign is likely to indicate rupture of the transverse humeral ligament. Such a rupture would be associated with which of the following? a. Rupture of the bicipital aponeurosis b. Avulsion of the coracoid process c. Dislocation of the tendon of the long head of the biceps brachii d. Tear of the glenoid labrum e. Subacromial bursitis

There is one degree of movement at the scapulothoracic joint

You are evaluating the scapulohumeral rhythm of your patient who has reduced shoulder movements. You know that for every 2 degree of elevation of the upper limb at the glenohumeral joint, which of the following is true? a. There is one degree of movement at the acromioclavicular joint b. There is one degree of movement at the scapulothoracic joint c. There are an equal number of degrees of movement at the acromioclavicular joint d. There are an equal number of degrees of movement at the scapulothoracic joint e. There are an equal number of degrees at sternoclavicular joint

To the inferior parts of the brachial plexus

You are reading a report of your 25-year-old patient who was in the ED and was diagnosed with Klumpke-Dejerine paralysis. This refers to injury to which of the following? a. To the inferior parts of the brachial plexus b. To the superior parts of the brachial plexus c. To the posterior cord of his brachial plexus d. To axillary nerve e. To his scalene muscles

Ulnar

You are testing the integrity of a nerve after an injury in a patient by holding the proximal interphalangeal joint of the little finger in the extended position and asking the patient to flex her distal interphalangeal joint. Which nerve is being tested? a. Median b. Radial c. Ulnar d. Musculocutaneous e. Axillary

AC joint dislocation ("shoulder separation")

You are treating a 28-year-old hockey player who jammed his shoulder into the boards while playing. His radiograph is shown above. What is your preliminary diagnosis? a. Anterior shoulder dislocation b. AC joint dislocation ("shoulder separation") c. Normal shoulder d. Posterior shoulder dislocation e. Glenoid labrum tear

Surgical neck

You are treating a patient in the ED with a fracture of the proximal end of the humerus. Most of these fractures occur at which of the following? a. Anatomical neck b. Surgical neck c. Lesser trochanter d. Greater trochanter e. Deltoid tuberosity

Fifth metacarpal fracture

Your 17-year-old male patient was in a fist fight in a bar and now has pain along the medial side of his distal hand, along with flexion deformity (caused by volar angulation of the distal fracture fragment). A radiograph of his hand is shown above. What is your diagnosis? a. Colles fracture b. Fracture of the triangular ligament c. Fifth metacarpal fracture d. Fracture of first phalange of fifth digit e. Dislocation of hamate-fifth metacarpal joint

Ask the patient to firmly hold a sheet a paper between extended fingers

Your patient is having some difficulty with hand movements and you suspect there may a spinal nerve lesion affecting his palmar interossei. To most effectively test these muscles specifically you would do which of the following? a. Ask the patient to spread his fingers against resistance b. Ask the patient to firmly hold a sheet a paper between extended fingers c. Ask the patient to extend the interphalangeal joints of his medial four fingers against resistance while keeping the metacarpal-phalangeal joints of those fingers flexed d. Ask the patient to extend his the metacarpal-phalangeal joints of his medial four fingers against resistance e. Ask the patient to flex his wrist against resistance

Extensor digiti Minimi

lateral epicondyle of humerus (common extensor origin), Extensor expansion of 5th finger, extends 5th finger primarily at metacarpophalangeal joint, secondarily at interphalangeal joint, Posterior Interosseous nerve (C7, C8), continuation of deep branch of radial nerve, Posterior interosseous artery

A depressed girl slashed her right wrist with a razor blade. She was rushed to the emergency department. The small spurts of blood from her wrist were difficult to stop. Physical examination of her wrist and hand revealed that a large nerve was severed. She could adduct, abduct, and extend her thumb but was unable to oppose it. She had lost some fine control of the movements of her second and third digits and there was loss of sensation over the lateral half of her palm and digits. Which of the following statements best describes the most likely cause of the signs resulting from severance of the nerve?

median nerve. This nerve is relatively superficial proximal to the wrist (especially in approximately 20% of the population lacking a palmaris longus). The nerve passes deep to the flexor retinaculum as it passes through the carpal tunnel to the hand. The nerve is motor to the intrinsic thumb muscles, except for the adductor pollicis and the deep head of the FPB, and to the lateral lumbricals for digits two and three. It is sensory to the skin of the palmar and distal dorsal aspects of the lateral three and a half digits and the adjacent palm. This explains why severance of the median nerve severely affects movements of the hand.

A physician tests the myotatic biceps reflex as shown. A normal response of involuntary contraction of the biceps brachii muscle is noted. This reflex confirms the integrity of what nerve?

muculocutaneous nerve. A positive response to the myotatic biceps reflex confirms the integrity of the musculocutaneous nerve and the C5 and C6 spinal segments, from which this nerve arises. The musculocutaneous nerve supplies motor innervation and proprioception to the muscles in the anterior compartment of the arm, including the coracobrachialis, biceps brachii (tested here), and brachialis. Lesion in the musculocutaneous nerve would lead to loss of proprioception and weakness in flexing the elbow (via the biceps brachii and brachialis muscles) and supinating the forearm (via the biceps brachii), resulting in a negative myotatic biceps reflex.

A 21-year-old male college student reports to the student health clinic on Monday morning, the day after the Super Bowl. He explains that he was intoxicated and lost consciousness with his upper limbs draped over the back of a couch. He complains of numbness and paresthesia over the dorsum of his hand on the radial side and is unable to support the weight of his left hand when the hand is placed in a pronated position (see photo). What nerve was most likely damaged in this individual?

radial nerve. This case represents a classic presentation of "Saturday Night Palsy," where the radial nerve is compressed against the humerus in the arm. Remember, the radial nerve supplies motor innervation to the posterior compartments of the arm and forearm, so damage to this nerve would cause weakness in extending the elbow and wrist. This patient is unable to extend the wrist when the hand is placed in a pronated position ("wristdrop"), implying damage to the radial nerve. Moreover, the superficial branch of the radial nerve is responsible for cutaneous innervation over much of the dorsum of the hand, which explains the numbness and paresthesia in his hand.

A 21-year-old man goes to his college campus health clinic complaining of soreness in his left wrist after falling on an outstretched hand during a basketball game the previous day. He is supporting his left wrist and indicates that the pain worsens with movement and is minimized with inactivity. There is no loss of feeling in his hand, nor does he have trouble grasping or holding objects. The physician exacerbates the wrist pain by applying pressure to the base of the thumb in the anatomical snuffbox. Radiographic imaging will confirm a break of which carpal bone?

scaphoid. The most commonly fractured carpal bone is the scaphoid that forms the floor of the anatomical snuffbox. This area is a fossa located between the three long tendons of the thumb (tendons of abductor pollicis longus and extensor pollicis brevis laterally and tendon of the extensor pollicis longus medially). The scaphoid bone is frequently broken when an individual falls with an outstretched hand and lands on the palm with the hand abducted. A broken scaphoid bone is commonly seen in individuals under the age of 30. The scaphoid is broken due to its unfortunate position between the downward force transmitted by the weight of the upper limb and the upward force due to the impact of hitting the floor. The proximal aspect of a broken scaphoid bone can suffer from avascular necrosis due to its blood supply entering the bone distally.

14-year-old boy fell off his skateboard, hitting his right elbow on the cement sidewalk. Because he was experiencing severe elbow pain, his mother took him to the hospital. On examination, the orthopedist concluded that his elbow was fractured. Radiographs revealed that there was a separation of the epiphysis of the medial epicondyle of the humerus. He noted that there was numbness and tingling of the medial part of the palm and the medial one and a half digits. Which nerve is most likely injured?

ulnar nerve injury. This injury commonly occurs where the nerve passes posterior to the medial epicondyle of the humerus. The nerve was probably sharply compressed resulting in bruising and crushing of nerve fibers as the epicondyle was broken away from the humerus. The medial palm and medial one and a half digits would be affected.


Set pelajaran terkait

Which of the following properties are chemical and which are physical?

View Set

AP Computer Science Essay Questions Assessment

View Set

CH 8: Therapeutic Communication

View Set

Chapter 22 : The Respiratory System

View Set

management final ch 1-3. multiple choice

View Set

OSCM Chapter 7: Manufacturing Processes

View Set